You are on page 1of 124

Engineering Mathematics I

Dr. V. Lokesha

2011

Engineering Mathematics I
(10 MAT11)

LECTURE NOTES
(FOR I SEMESTER B E OF VTU)

VTU-EDUSAT Programme-15

Dr. V. Lokesha Professor and Head DEPARTMENT OF MATHEMATICS ACHARYA INSTITUTE OF TECNOLOGY Soldevanahalli, Bangalore 90
10 MAT11 1

Engineering Mathematics I

Dr. V. Lokesha

2011

ENGNEERING MATHEMATICS I Content


CHAPTER
UNIT I UNIT II UNIT III DIFFERENTIAL CALCULUS I DIFFERENTIAL CALCULUS II DIFFERENTIAL CALCULUS III

10 MAT11 2

Engineering Mathematics I

Dr. V. Lokesha

2011

UNIT - I
DIFFERENTIAL CALCULUS I Introduction:
The mathematical study of change like motion, growth or decay is calculus. The Rate of change of given function is derivative or differential. The concept of derivative is essential in day to day life. Also applicable in Engineering, Science, Economics, Medicine etc.

Successive Differentiation:
Let y = f (x) --(1) be a real valued function. dy The first order derivative of y denoted by or y or y1 or 1 dx 2 The Second order derivative of y denoted by d yor y or y2 or 2 dx 2 Similarly differentiating the function (1) n-times, successively, dny th the n order derivative of y exists denoted by n or yn or yn or n dx The process of finding 2nd and higher order derivatives is known as Successive Differentiation.

nth derivative of some standard functions:


1. Sol : y = eax y1 = a eax y2 = a2 eax
Differentiating Successively

yn = an eax ie. Dn[eax] = an eax For, a =1 Dn[ex] = ex


10 MAT11 3

Engineering Mathematics I

Dr. V. Lokesha

2011

10 MAT11 4

Engineering Mathematics I

Dr. V. Lokesha

2011

10 MAT11 5

Engineering Mathematics I

Dr. V. Lokesha

2011

10 MAT11 6

Engineering Mathematics I

Dr. V. Lokesha

2011

10 MAT11 7

Engineering Mathematics I

Dr. V. Lokesha

2011

Leibnitzs Theorem :
It provides a useful formula for computing the nth derivative of a product of two functions. Statement : If u and v are any two functions of x with un and vn as their nth derivative. Then the nth derivative of uv is

(uv)n = u0vn + nC1 u1vn-1 + nC2u2vn-2 + +nCn-1un-1v1+unv0


Note : We can interchange u & v (uv)n = (vu)n,
nC = 1

n , nC2 = n(n-1) /2! , nC3= n(n-1)(n-2) /3!

1.

Find the nth derivations of eax cos(bx + c) Solution: y1 = eax b sin (bx +c) + a eax cos (b x + c), by product rule. .i.e, y1 = eax [a cos (bx + c ) b sin (bx + c )] Let us put a = r cos , and b = r sin .

a 2 + b 2 = r 2 and tan = b / a
.ie., r = a 2 + b 2 and = tan-1 (b/a) Now, y1 = e ax [r cos cos(bx + c) r sin sin( bx + c)] Ie., y1 = r eax cos ( + bx + c ) where we have used the formula cos A cos B sin A sin B = cos (A + B) Differentiating again and simplifying as before, y2 = r2 eax cos (2 + bx + c ) . Similarly y3 = r3 e ax cos (3 + bx + c ) . Thus y n = r n e ax cos(n + bx + c ) Where r = a 2 + b 2 and = tan-1 (b/a). Thus Dn [eax cos (b x + c)] = ( a 2 + b 2 ) n e ax cos n tan 1 (b / a ) + bx + c

]]

10 MAT11 8

Engineering Mathematics I

Dr. V. Lokesha

2011

2. Find the nth derivative of log Solution : Let y = log ie., y =


y=

4x2 + 8x + 3

4 x 2 + 8x + 3 = log (4x2 + 8x +3)

1 log (4x2 + 8x +3) log xn = n log x 2

1 log { (2x + 3) (2x+1)}, by factorization. 2 1 {log (2x + 3) + log (2x + 1)} 2


n 1 n 1

y=

1 ( 1) (n 1)!2 n ( 1) (n 1)!2 n + Now y n = 2 (2x + 3)n (2x + 1)n


1 1 + Ie., yn = 2n-1 (-1) n-1 (n-1) ! n n (2 x + 3) (2 x + 1)
3. Find the nth derivative of log 10 {(1-2x)3 (8x+1)5} Solution : Let y = log 10 {1-2x)3 (8x+1)5} It is important to note that we have to convert the logarithm to the base e by the property:
log10 x = log e x log e 10

Thus y = Ie., y =

1 3 5 log e (1 2 x ) (8x + 1) log e 10

1 {3 log(1 2x ) + 5 log(8x + 1)} log e 10


n 1 n ( 1)n 1 (n 1)!8n 1 ( 1) (n 1)!( 2) 3 . 5 + loge 10 (1 2x )n (8x + 1)n

yn =

Ie.,

n 1 n n ( 5(4) 1) (n 1)!2 n 3( 1) yn = + n n log e 10 (1 2x ) (8x + 1)

10 MAT11 9

Engineering Mathematics I

Dr. V. Lokesha

2011

4. Find the nth derivative of e2x cos2 x sin x


1 + cos 2 x Solution : >> let y = e2x cos2 x sin x = e2x sin x 2

ie., y = =

e2x (sin x + sin x cos 2x) 2


1 sin x + [sin 3x + sin ( x )] 2

e2x 2

e2x (2 sin x + sin 3x sin x ) sin (-x) = -sin x = 4


e2x y = (sin x + sin 3x) 4
Now y n = Thus y n =
1 n 2x D e sin x + D n e 2 x sin 3x 4 1 4

{ (

)}

{( 5 ) e
n

2x

sin n tan 1 (1 2 ) + x +
1

] ( 13 ) e
n

2x

sin n tan 1 (3 2 ) + 3x

]}

yn =

e2x 4

{( 5 ) sin[n tan
n

(1 2) + x ]+ (

13 sin n tan 1 (3 2) + 3x
n

) [

]}

5. Find the nth derivative of e2x cos 3x Solution : Let y=e2x cos3 x = e 2x. Ie.,
yn = yn = 1 (3 cos x + cos 3x) 4

y=

1 (3 e2x cos x + e2x cos 3x) 4

n 1 3 5 e 2 x cos n tan 1 (1 2 ) + x + 4

{( )

1 {3Dn (e2x cos x) + Dn (e2x cos 3x)} 4

] ( 13 ) e
n

2x

cos n tan 1 (3 2 ) + 3 x

]}

Thus y n =

n n e2x 3 5 cos n tan 1 (1 2) + x + 13 cos n tan 1 (3 2) + 3x 4

{( )

] ( )

]}

10 MAT11 10

Engineering Mathematics I

Dr. V. Lokesha

2011

6. Find the nth derivative of Solution :


y=

x2 (2x + 1)(2x + 3)

x2 is an improper fraction because; the degree of the (2x + 1)(2x + 3)

numerator being 2 is equal to the degree of the denominator. Hence we must divide and rewrite the fraction.

y=

4x 2 x2 1 = for convenience. . 4 x 2 + 8x + 3 4 4 x 2 + 8x + 3

4x
4x2 +8x +3

4 x 2 + 8x + 3 8x 3

Ie., y =

y=

1 8x 3 1+ 2 4 4 x + 8x + 3

1 1 8x + 3 2 4 4 4 x + 8x + 3 1 8x + 3 yn = 0 Dn 2 4 4 x + 8x + 3

The algebraic fraction involved is a proper fraction. Now Let

8x + 3 A B = + (2x + 1)(2x + 3) 2x + 1 2x + 3
................(1)

Multiplying by (2x + 1) (2x + 3) we have, 8x + 3 = A (2x + 3) + B (2x + 1) By setting 2x + 1 = 0, 2x + 3 = 0 we get x = -1/2, x = -3/2. Put x = -1/2 in (1): -1 -1 + A (2) A = -1/2 Put x = -3/2 in (1): -9 = B (-2) B = 9/2

1 1 1 9 n 1 y n = D n + D 4 2 2 x + 1 2 2x + 3
10 MAT11 11

Engineering Mathematics I

Dr. V. Lokesha

2011

( 1)n n!2n + 9 ( 1)n n!2n 1 = ( 1) 8 (2x + 3)n +1 (2x + 1)n +1


ie., y n
n +1 ( 1) n!2 n =

1 9 + n +1 n +1 (2x + 3) (2x + 1)

7. Find the nth derivative of Solution : y =

x4 ( x + 1) ( x + 2)

x4 is an improper fraction. ( x + 1) ( x + 2)

(deg of nr. = 4 > deg. of dr. = 2) On dividing x4 by x2 + 3 x + 2, We get


15 x 14 y = ( x2 3x + 7 ) + 2 x + 3x + 2

15 x 14 yn = Dn (x2-3x+7)-Dn 2 x + 3x + 2

But D = ( x2 3x + 7 ) = 2x 3, D2 ( x2 3x + 7 ) = 2 D3( x2 3x + 7 ) = 0......... Dn ( x2 3x + 7 ) = 0 if n > 2

15 x + 14 Hence yn = -Dn ( x + 1) ( x + 2)
Now, let Dn

A B 15 x + 14 = + 2 x + 3x + 2 ( x + 1) ( x + 2)

=> 15x+ 14 = A(x+2) + B(x+ 1 ) Put x = - 1 ; - 1 = A ( 1 ) or A = - 1 Put x = - 2 ; - 16 = B ( - 1 ) or B = 16

1 1 + 16 D n Yn = D n x + 2 x + 1

10 MAT11 12

Engineering Mathematics I
= (1) n n ! 1n (1) n n ! 1n 16 ( x + 1) n +1 ( x + 2) n +1

Dr. V. Lokesha

2011

1 16 yn = (1) n n ! n> 2 n +1 ( x + 2) n +1 ( x + 1)

8. Show that
d n log x (1) n n! 1 1 1 = log x 1 n n +1 2 3 n dx x x

Solution : Let y =

log x 1 1 = log x. and let u = log x, v = x x x

We have Leibnitz theorem, (uv)n = uvn + nC1 u1v n 1 + nC2 u 2 v n 2 + .... + u n v Now, u = log x (1)

(1) n1 (n 1)! un = xn

( 1) n n! 1 v = vn = x n+1 x
Using these in (1) by taking appropriate values for n we get,
(1) n n! 1 (1) n1 (n 1)! log x Dn = = log x.. n +1 + n . x x xn x

n(n 1) 1 (1) n2 (n 2)! 1. 2 x 2 x n 1 + ...... + (1) n1 (n 1)! 1 . x xn

Ie.. = log x

(1) n n! (1) n1 n! + x n+1 x n+1

(1) n 2 n! (1) n 1 (n 1)! + .... + x n +1 2 x n +1

(1) n2 n! (1) 2 (1) 1 (n 1)! 1 + .... + log x(1) x n+1 2 n1


Note : (-1)-1 =

1 1 = 1; (1) 2 = =1 1 (1) 2
10 MAT11 13

Engineering Mathematics I Also

Dr. V. Lokesha

2011

(n 1)! (n 1)! 1 = = n! n (n 1)! n

1 1 1 d n log x (1) n n! log x 1 ... = n n +1 2 3 n dx x x

9. If yn= Dn (xn logx) Prove that yn = n yn-1+(n-1)! and hence deduce that
1 1 1 yn = n log x + 1 + + + .... + 2 3 n

Solution : yn = Dn(xn log x) = Dn-1 {D (xn log x}


n 1 n 1 = Dn-1 x . + nx log x x

= Dn-1(xn-1) + nDn-1 (xn-1 log x}

yn = (n-1)! +nyn-1. This proves the first part.


Now Putting the values for n = 1, 2, 3...we get y1 = 0! + 1 y0 = 1 + log x = 1! (log x + 1 ) y2 = l! + 2y1 = l+2 (l + log x)
1 ie., y2 = 21og x + 3 = 2(log x + 3/2) = 2! log x + 1 + 2

y3 = 2! + 3y2 = 2 + 3(2 log x + 3)


1 1 ie., y3 = 61og x+ll = 6 (log x + ll/6) = 3! log x + 1 + + 2 3

..
1 1 1 y n = n! log x + 1 + + + ... + n 2 3

10. If y = a cos (log x) + b sin ( log x), show that x2y2 + xy1 + y = 0. Then apply Leibnitz theorem to differentiate this result n times. or If y = a cos (log x) + b sin (log x ), show that x2yn + 2 + (2n+l)xyn + l+(n2+1)yn = 0. [July-03] 10 MAT11 14

Engineering Mathematics I Solution : y = a cos (log x) + b sin (log x) Differentiate w.r.t x

Dr. V. Lokesha

2011

y1 = -a sin (log x)

1 1 + b cos (log x). x x

(we avoid quotient rule to find y2) . => xy1 = - a sin (log x) + b cos (log x) Differentiating again w.r.t x we have, xy2 + 1 y1 = - a cos (log x) + b sin ( log x) or
1 x

x2y2 + xy1 = - [ a cos (log x) + b sin (log x) ] = -y

x2y2+xy1+y = 0
Now we have to differentiate this result n times. ie., Dn (x2y2) + Dn (xy1) + Dn (y) = 0 We have to employ Leibnitz theoreom for the first two terms. Hence we have,

2 n n(n 1) n 1 . 2 . D n2 ( y 2 ) ) x . D ( y 2 ) + n. 2 x. D ( y 2 ) + 1. 2

{x. D

( y1 ) + n. 1 . D n 1 ( y1 ) + y n = 0

ie., {x2yn + 2 + 2n x yn + 1 + n (n 1)yn} + {xyn+1+nyn}+yn = 0 ie., x2yn + 2 + 2n x yn + 1 + n2yn - nyn + xyn+1+nyn+yn = 0 ie., x2yn + 2 + (2n+l)xy n+l + (n2+l)yn = 0

11. If cos-1 (y/b ) = log (x/n)n, then show that x2yn + 2 + (2n+l) xy n+l + 2n2yn = 0 Solution :By data, cos-1 (y/b) = n log (x/n) log(am) = m log a =>
y = cos [n log (x/n )] b

or y = b . cos [ n log (x/n)] Differentiating w.r.t x we get, 10 MAT11 15

Engineering Mathematics I y1 = -b sin [n log (x/n)] n

Dr. V. Lokesha

2011

1 1 (x / n ) n

or xy1 = - n b sin [n log (x/n )] Differentiating w.r.t x again we get, xy2 + 1. y1 = - n . b cos [ n log (x/n )] n

1 1 . ( x / n) n

or x (xy2+y1) = n2b cos [n log (x/n) ] =-n2y, by using (1). or x2y2 +xyl + n2y = 0 Differentiating each term n times we have, D(x2y2) + Dn(xy1) + n2Dn (y) = 0 Applying Leibnitz theorem to the product terms we have,
2 n(n 1) . 2 . yn x y n + 2 + n. 2 x. y n +1 + 1. 2 2 + {xy n +1 + n. 1 . y n }+ n y n = 0

ie x2yn+2 + 2 x yn+1 + n2yn + xy n+1+ nyn + n2yn=0 or x2 yn+2 + (2n + l) xyn+1 + 2n2yn = 0 12. If y = sin( log (x2 + 2 x + 1)), or If sin-1 y = 2 log (x + 1), show that (x+l)2yn + 2 + (2n+1)(x+1)yn+l + (n2 + 4)yn = 0 Solution : By data y = sin log (x2 + 2 x + 1 ) [Feb-03]

y1 = cos log (x2 + 2 x + 1)


ie., y1 = cos log (x2 + 2 x + 1)

1 2x + 2 ( x + 1) 2
1 2 (x + 1) x + 2x +1
2

2 cos log( x 2 + 2 x + 1 ) ie., y1 = ( x + 1)

or (x + 1) y1 = 2 cos log (x2 + 2 x + 1 ) Differentiating w.r.t x again we get

10 MAT11 16

Engineering Mathematics I (x+1)y2 + 1 y1 = -2 sin log (x2 + 2x + 1) or (x + 1)2y2 + (x+1) y1 = -4y or (x+l )2y2 + (x+l) y1 + 4y = 0 , Differentiating each term n times we have, Dn [(x + 1)2y2] +Dn [(x+ 1)y1] + Dn [y] = 0 Applying Leibnitz theorem to the product terms we have,

Dr. V. Lokesha

2011

1 . 2( x + 1) ( x + 1) 2

n(n 1) 2 .2 . y n ( x + 1) y n+ 2 + n. 2( x + 1). y n+1 + 1. 2


+ {(x+l) yn + 1+n. 1 .yn} + 4yn = 0 ie., ie., (x+l)2yn + 2 + 2n (x+1)yn+1 + n2yn-nyn + (x+l)yn+l + nyn + 4yn = 0 (x+l)2yn + 2 + (2n + l) (x + l) y n+ 1 + (n2 + 4)yn = 0

13. If = log x + 1 + x 2 prove that (1 + x2) yn+2 + (2n + 1) xyn+1 + n2yn = 0 >> By data, y = log x + 1 + x 2

y1 =

1 1 + . 2 x 2 2 ( x + 1+ x ) 2 1+ x 1

Ie., y1 or

1 ( x + 1+ x )
2

1+ x2 + x 1+ x
2

1 1+ x2

1 + x 2 y1 = 1

Differentiating w.r.t.x again we get

1 + x 2 y2 +

1 2 1+ x 2 )

.2 x. y1 = 0

or (1+x2)y2 + xy1 = 0 Now Dn [(l+x2)y2] + Dn[xy1] = 0 Applying Leibnitz theorem to each term we get, 10 MAT11 17

Engineering Mathematics I

Dr. V. Lokesha

2011

n(n 1) 2 .2 . y n (1 + x ) y n+ 2 + n. 2 x . y n+1 + 1 .2
+ [x . yn + 1+n .1 yn] = 0 Ie., (1 + x2) yn +2 + 2 n x yn + 1 + n2yn nyn + xyn+l+ nyn = 0 or (l+x2)yn + 2 + (2n + l)xyn+1+n2yn = 0 14. If x = sin t and y = cos mt, prove that (l-x2)yn + 2-(2n+1)xyn+l + (m2-n2)yn = 0. Solution : By data x = sin t and y = cos mt x = sin t => t = sin-1 x and y = cos mt becomes y = cos [ m sin-1x) Differentiating w.r.t.x we get y1 = - sin (m sin-1x)
m

[Feb-04]

1 x2

or 1 + x 2 y1 = - m sin (m sin-1x) Differentiating again w.r.f .x we get,


1 x 2 y2 + 1 2 1 x
2

(2 x ) y1 = m cos (m sin 1 x ).

1 x2

or (1 -x2)y2-xyl = -m2y or (1 -x2)y2 xy1 +m2y = 0 Thus (1-x2)yn+2-(2n+1)xyn+1+(m2-n2)yn=0 15. If x = tan ( log y), find the value of (l+x2)yn+1 + (2nx-l) yn+n(n-1)yn-1 [July-04] Solution : By data x = tan(log y) => tan-1 x = log y or y = etan-1 x Since the desired relation involves yn+1, yn and yn-1 we can find y1 and differentiate n times the result associated with y1 and y. Consider y = e tan or (1 +x2)y1 = y Differentiating n times we have 10 MAT11 18
1

y. = e tan

1 1+ x2

Engineering Mathematics I Dn[(l+x2)y1]=Dn[y] Anplying Leibnitz theorem onto L.H.S, we have, {(l+x2)Dn(y1) + n .2x .Dn-1 (y1)
+ n( n 1) .2 .D n 2 ( y1 )} = y n 1 .2

Dr. V. Lokesha

2011

Ie., (1+x2)yn+1+2n x yn + n (n-1) yn-1-yn=0 Or (l+x2)yn + 1 + (2nx-l)yn + n(n-l)yn-1 = 0

10 MAT11 19

Engineering Mathematics I

Dr. V. Lokesha

2011

Continuity & Differentiability


Some Fundamental Definitions

A function f (x) is defined in the interval I, then it is said to be continuous at a point x = a f ( x) = f (a) if lim xa f ( x + h) f (a) = f '(a) exists a I A function f (x) is said to be differentiable at x = a if lim h0 h
Ex : Consider a function f (x) is defined in the interval [-1,1] by f (x) =
x x = x 1 x 0 0 x 1

It is continuous at x = 0 But not differentiable at x = 0 Note : If a function f (x) is differentiable then it is continuous, but converse need not be true. Geometrically : (1) If f (x) is Continuous at x =a means, f (x) has no breaks or jumps at the point x = a Ex :
1 f (x) = x 1 x 0 0 < x 1

Is discontinuous at x=0 (2) If f (x) is differentiable at x = a means, the graph of f (x) has a unique tangent at the point or graph is smooth at x = a 1. Give the definitions of Continuity & Differentiability: Solution: A function f (x) is said to be continuous at x = a, if corresponding to an arbitrary positive number , however small, their exists another positive number such that. f (x) f (a) < , where x - a < It is clear from the above definition that a function f (x) is continuous at a point a. If (i) it exists at x = a (ii) Lt f (x) = f (a)
xa

i.e, limiting value of the function at x = a is to the value of the function at x = a 10 MAT11 20

Engineering Mathematics I

Dr. V. Lokesha

2011

Differentiability: A function f (x) is said to be differentiable in the interval (a, b), if it is differentiable at every point in the interval. In Case [a,b] the function should posses derivatives at every point and at the end points a & b i.e., Rf1 (a) and Lf1 (a) exists. 2. State Rolles Theorem with Geometric Interpretation. Statement: Let f (x) be a function is defined on [a,b] & it satisfies the following Conditions. (i) (ii) f (x) is continuous in [a,b] f (x) is differentiable in (a,b)

(iii) f (a) = f (b) Then there exists at least a point C (a,b), Here a < b such that f1 ( c ) = 0 Proof: Geometrical Interpretation of Rolles Theorem: Y y = f (x) P A f(a) Q S f(a) x=a x=c x=b f(b) R B A B

O x =a c1 c2 c3 c4 x = b

Let us consider the graph of the function y = f (x) in xy plane. A (a,.f(a)) and B (b, f( b ) ) be the two points in the curve f (x) and a, b are the corresponding end points of A & B respectively. Now, explained the conditions of Rolles theorem as follows. (i) (ii) f (x) is continuous function in [a,b], Because from figure without breaks or jumps in between A & B on y = f (x). f (x) is a differentiable in (a,b), that means let us joining the points A & B, we get a line AB.

Slope of the line AB = 0 then a point C at P and also the tangent at P (or Q or R or S) is Parallel to x axis. Slope of the tangent at P (or Q or R or S) to be Zero even the curve y = f (x) decreases or increases, i.e., f (x) is Constant. 10 MAT11 21

Engineering Mathematics I

Dr. V. Lokesha

2011

f1 (x) = 0 f1 (c) = 0 (iii) The Slope of the line AB is equal to Zero, i.e., the line AB is parallel to x axis. f (a) = f (b) 3. Verify Rolles Theorem for the function f (x) = x2 4x + 8 in the internal [1,3] Solution: We know that every Poly nominal is continuous and differentiable for all points and hence f (x) is continuous and differentiable in the internal [1,3]. Also f (1) = 1 4 + 8 = 5, f (3) = 32 43 + 8 = 5 Hence f (1) = f (3) Thus f (x) satisfies all the conditions of the Rolles Theorem. Now f1 (x) = 2x 4 and f1 (x) = 0 2x 4 = 0 or x = 2. Clearly 1 < 2 < 3. Hence there exists 2t (1,3) such that f1 (2) = 0. This shows that Rolles Theorem holds good for the given function f (x) in the given interval. 4. Verify Rolles Theorem for the function f (x) = x (x + 3) e Solution: Differentiating the given function W.r.t x we get
x 1 x f 1 ( x) = ( x 2 + 3 x) e 2 + (2 x + 3)e 2 2 1 x = ( x 2 x 6) e 2 2 1 f (x) exists (i.e finite) for all x and hence continuous for all x.

in the interval [-3, 0]

Also f (-3) = 0, f (0) = 0 so that f (-3) = f (0) so that f (-3) = f (0). Thus f (x) satisfies all the conditions of the Rolles Theorem. Now, f1 (x) = 0
1 x ( x 2 x 6) e 2 = 0 2 Solving this equation we get x = 3 or x = -2

Clearly 3 < -2 < 0. Hence there exists 2 (-3,0) such that f1 (-2) = 0 This proves that Rolles Theorem is true for the given function.

10 MAT11 22

Engineering Mathematics I

Dr. V. Lokesha

2011

5.

Verify the Rolles Theorem for the function Sin x in [-, ]

Solution: Let f (x) = Sin x Clearly Sinx is continuous for all x. Also f1 (x) = Cos x exists for all x in (-, ) and f (-) = Sin (-) = 0; f () = Sin () = 0 so that f (-) = f () Thus f (x) satisfies all the conditions of the Rolles Theorem . Now f1 (x) = 0 Cos x = 0 so that X=

Both these values lie in (-,). These exists C = Such that ( c ) = 0 f1

Hence Rolles theorem is vertified.


6. Discuss the applicability of Rolles Theorem for the function f (x) = x in [-1,1]. Solution: Now f (x) = x= x for 0 x 1 -x for 1 x 0

f (x) being a linear function is continuous for all x in [-1, 1]. f(x) is differentiable for all x in (1,1) except at x = 0. Therefore Rolles Theorem does not hold good for the function f (x) in [-1,1]. Graph of this function is shown in figure. From which we observe that we cannot draw a tangent to the curve at any point in (-1,1) parallel to the x axis. Y y = x

-1

10 MAT11 23

Engineering Mathematics I

Dr. V. Lokesha

2011

Exercise: 7. Verify Rolles Theorem for the following functions in the given intervals. a) x2 6x + 8 in [2,4] b) (x a)3 (x b)3 in [a,b]

x 2 + ab c) log in [a,b] ( a + b) x 8. Find whether Rolles Theorem is applicable to the following functions. Justify your answer.
a) f (x) = x 1 in [0,2] b) f (x) = tan x in [0, ] . 9. State & prove Lagranges (1st) Mean Value Theorem with Geometric meaning. Statement: Let f (x) be a function of x such that (i) If is continuous in [a,b] (ii) If is differentiable in (a,b) Then there exists atleast a point (or value) C (a,b) such that. f (b) f (a ) f 1 (c ) = ba i.e., f (b) = f (a) + (b a) f1 (c) Proof:

y [b,f(b)] [a,f(a)] x a c b
10 MAT11 24

Engineering Mathematics I

Dr. V. Lokesha

2011

Define a function g (x) so that g (x) = f (x) Ax ---------- (1) Where A is a Constant to be determined. So that g (a) = g (b) Now, g (a) = f (a) Aa G (b) = f (b) Ab g (a) = g (b) f (a) Aa = f (b) Ab.
f (b) f ( a ) ---------------- (2) ba Now, g (x) is continuous in [a,b] as rhs of (1) is continuous in [a,b] G(x) is differentiable in (a,b) as r.h.s of (1) is differentiable in (a,b).

i.e., A =

Further g (a) = g (b), because of the choice oif A. Thus g (x) satisfies the conditions of the Rolles Theorem. These exists a value x = c sothat a < c < b at which g1 ( c ) = 0 Differentiate (1) W.r.t x we get g1 (x) = f 1 (x) A g1 ( c ) = f1 ( c )- A ( x =c) f 1 ( c ) - A = 0 (g1 ( c ) = 0) f1 ( c ) = A -------------- (3)

From (2) and (3) we get


f 1 (c ) = f (b) f (a ) (or) f (b) = f (a) + (b a) f1 (c) For a < c < b ba

Corollary: Put b a = h i.e., b = a + h and c = a + h Where 0 < < 1 Substituting in f (b) = f (a) + (b a) f1 ( c ) f (a + h) = f (a) + h f (a + h), where 0 < < 1. 10 MAT11 25

Engineering Mathematics I

Dr. V. Lokesha

2011

Geometrical Interpretation:Since y = f (x) is continuous in [a,b], it has a graph as shown in the figure below, At x = a, y = f (a) At x = b, y = f (b)

Y A C

Y P A Q x

X 0 Figure (ii)

c Figure (i)

Slope of the line joining the points A (a,f(a)) and B ( (b,f (b)) Is
f (b) f (b) ba

( Slope = m = tan )

= tan Where is the angle mode by the line AB with x axis = Slope of the tangent at x = c = f1 ( c ), where a < c < b Geometrically, it means that there exists at least are value of x = c, where a < c < b at which the tangent will be parallel to the line joining the end points at x = a & x = b. Note: These can be more than are value at which the tangents are parallel to the line joining points A & B (from Fig (ii)).

10 MAT11 26

Engineering Mathematics I

Dr. V. Lokesha

2011

10. Verify Lagranges Mean value theorem for f(x) = (x 1) (x 2) (x 3) in [0,4]. Solution: Clearly given function is continuous in [0,4] and differentiable in (0,4), because f (x) is in polynomial. f (x) = (x 1) (x 2) (x 3) f (x) = x3 6x2 + 11x 6 and f (0) = 03 6(0)2 + 11 (0) 6 = -6 f(4) = 43 6 (4)2 + 11 (4) 6 = 6 Differentiate f (x) W.r.t x, we get F1 (x) = 3x2 6x + 11 Let x = c, f1 ( c) = 3c2 6c + 11 By Lagranges Mean value theorem, we have

f (b) f (a) f (4) f (0) = ba (4 0) 6 ( 6) = =3 4 f 1 (c ) =


3c2 6c + 11 = 3 3c2 6c + 8 = 0 Solving this equation, we get (0,4) 3 Hence the function is verified. 11. Verify the Lagranges Mean value theorem for f (x) = logx in [1,e]. Solution: Now Logx is continuous for all x > 0 and hence [1,e]. 1 Also f 1 ( x) = which exists for all x in (1,e) x Hence f (x) is differentiable in (1,e) by Lagranges Mean Value theorem, we get 10 MAT11 27 C=2

Engineering Mathematics I
Loge Log1 1 1 1 = = e 1 c e 1 c C=e1

Dr. V. Lokesha

2011

1<e- <2<e Since e (2,3) So that c = e 1 lies between 1 & e Hence the Theorem. 12. Find for f (x) = Lx2 + mx + n by Lagranges Mean Value theorem. Solution: f (x) = Lx2 + mx + n f1 (x) = 2 Lx + m We have f (a + h) = f (a) + hf1 (a + h) Or f (a + h) f (a) = hf1 (a + h) i.e., { (a + h)2 + m (a + h) + n} { a2 + ma + n} = h {2 (a + h) + m} Comparing the Co-efficient of h2, we get 1 = 2 Exercise: 13. 14. =
1 (0,1) 2

15.

Verify the Lagranges Mean Value theorem for f (x) = Sin2x in 0, 2 ba ba Prove that, < tan-1 b tan-1 a < if 0 < a < b and reduce that 2 1+ b 1+ a2 3 4 1 + < tan 1 < + 4 25 3 4 6 2 Sinx Show that < < 1 in 0, x 2

16. Prove that

ba

1 a2 1 b2 1 1 1 < Sin 1 < . 6 2 3 4 6 15

< Sin 1b Sin 1 a <

ba

Where a < b. Hence reduce

10 MAT11 28

Engineering Mathematics I

Dr. V. Lokesha

2011

17. State & prove Cauchys Mean Value Theorem with Geometric meaning. Proof: The ratio of the increments of two functions called Cauchys Theorem. Statement: Let g (x) and f (x) be two functions of x such that, (i) Both f (x) and g (x) are continuous in [a,b] (ii) Both f (x) and g (x) are differentiable in (a,b) (iii) g1 (x) 0 for any x (a,b) These three exists at least are value x = c (a,b) at which

f 1 (c) f (b) f (a) = g 1 (c) g (b) g (a )


Proof: Define a function, (x) = f (x) A. g (x) ------------------ (1)

So that (a) = (b) and A is a Constant to be determined. Now, (a) = f (a) A g (a) (b) = f (b) A. g (b) f (a) A g (a) = f (b) A. g (b) f (b) f (b) -------------------- (2) A= g (b) g (a ) Now, is continuous in [a,b] as r.h.s of (1) is continuous in [a,b] and (x) is differentiable in (a,b) as r.h.s of (1) is differentiable in [a,b]. Also (a) = (b) Hence all the conditions of Rolles Theorem are satisfied then there exists a value x = c (a,b) such that 1 ( c ) = 0. Now, Differentiating (1) W.r.t x, we get 1 (x) = f1 (x) A.g1 (x) at x = c (a,b) 10 MAT11 29

Engineering Mathematics I

Dr. V. Lokesha

2011

1 ( c ) = f1 ( c ) A g1 (c) 0 = f1 ( c) A g1 ( c ) A= ( g1 (x) 0)

f 1 (c ) --------------- (3) g 1 (c ) Substituting (3) in (2), we get f 1 (c) f (b) f (a) , where a < c < b = g 1 (c) g (b) g (a ) Hence the proof.
18. Verify Cauchys Mean Value Theorem for the function f (x) = x2 + 3, g (x) = x3 + 1 in [1,3] Solution: Here f (x) = x2 + 3, g (x) = x3 + 1 Both f (x) and g (x) are Polynomial in x. Hence they are continuous and differentiable for all x and in particular in [1,3] Now, f1 (x) = 2x, g1 (x) = 3x2 Also g 1 (x) 0 for all x (1,3)

Hence f (x) and g (x) satisfy all the conditions of the cauchys mean value theorem. Therefore f (3) f (1) f 1 (c) , for some c : 1 < c < 3 = g (3) g (1) g 1 (c)
i.e.,
12 4 26 = 28 2 3c 2 2 1 13 1 = C = =2 13 3c 6 6 1 lies between 1 and 3. 6

i.e.,

Clearly C = 2

Hence Cauchys theorem holds good for the given function.

10 MAT11 30

Engineering Mathematics I

Dr. V. Lokesha

2011

19. Verify Cauchys Mean Value Theorem for the functions f (x) Sin x, g (x) = Cos x in 0, 2

Solution: Here f (x) = Sin x, g (x) = Cos x so that f1 (x) = Cos x ,g1 (x) = - Sinx
Clearly both f (x) and g (x) are continuous in 0, , and differentiable in 0, 2 2

Also g1 (x) = -Sin x 0 for all x 0,

From cauchys mean value theorem we obtain

f f (0) f 1 (c ) 2 = 1 for some C : 0 < C < 2 g (c ) g g (0) 2


i.e.,
1 0 Cosc = 0 1 Sinc

i.e., -1 = - Cot c (or) Cot c = 1

C=

, clearly C =

lies between 0 and

Thus Cauchys Theorem is verified. Exercises: 20. Find C by Cauchys Mean Value Theorem for a) f (x) = ex, g (x) = e-x in [0,1] b) f (x) = x2, g (x) = x in [2,3] 21. Verify Cauchys Mean Value theorem for

1 a) f (x) = tan-1 x, g (x) = x in ,1 3 1 b) f(x) = log x, g (x) = in [1,e] x


10 MAT11 31

Engineering Mathematics I

Dr. V. Lokesha

2011

Generalized Mean Value Theorem: 22. State Taylors Theorem and hence obtain Maclaurins expansion (series)
Statement: If f (x) and its first (n 1) derivatives are continuous in [a,b] and its nth derivative exists in (a,b) then

(b a ) n 1 n-1 (b a) n n (b a) 2 11 f (a) + ---------+ f (a) + f(b) = f(a) + (b a) (a) + f (c ) (n 1)! n! 2!


f1 Where a < c < b Remainder in Taylors Theorem: We have f (x) = f (a) + (x a) f 1 (a) +

( x a) 2 11 ( x a) n 1 n-1 f (a) + --------- + f (a) + 2! (n 1)!

( x a) n n f [a + (x a) } n
f (x) = S n (x) + R n (x)

( x a) n n f [a + (x a) ] is called the Largranges form of the Remainder. n! xn n Where a = 0, R n (x) = f (x), 0 < < 1 n! Taylors and Maclaurins Series:
Where R n (x) = We have f (x) = Sn (x) + Rn (x)

Lim[ f ( x ) S n ( x )] = Lim Rn ( x )
n n

If Lim Rn ( x) = 0thenf ( x) Lim S n ( x )


n n

Thus Lim S n ( x) converges and its sum is f (x).


n

This implies that f (x) can be expressed as an infinite series. i.e., f (x) = f (a) + (x a) f 1 (a) + This is called Taylors Series.

( x a) 2 11 f (a) + ---------- to 2!

10 MAT11 32

Engineering Mathematics I

Dr. V. Lokesha

2011

Putting a = 0, in the above series, we get ( x) 2 11 F (x) = f (0) + x f 1 (0) + f (0) + --------- to 2! This is called Maclaurins Series. This can also denoted as ( x) 2 ( x) n Y = y (0) + x y 1 (0) + y 2 (0) + -------- + y n (0) ---------- to 2! n! Where y = f (x), y1 = f 1 (x), -------------- y n = f n (x) 23. By using Taylors Theorem expand the function e x in ascending powers of (x 1) Solution: The Taylors Theorem for the function f (x) is ascending powers of (x a) is ( x a ) 2 11 f (x) = f (a) + (x a) f 1 (a) + f (a) + ------------ (1) 2 Here f (x) = e x and a = 1 f 1 (x) = e x f 1 (a) = e f 11 (x) = e x f 11 (a) = e

(1) becomes ( x 1) 2 e = e + (x 1) e + e + ------------2 ( x 1) 2 = e { 1 + (x 1) + + --------} 2


x

24. By using Taylors Theorem expand log sinx in ascending powers of (x 3) Solution: f (x) = Log Sin x, a = 3 and f (3) = log sin3 Now f 1 (x) =
Cosx = Cotx, f 1 (3) = Cot3 Sinx

f 11 (x) = - Cosec 2x, f 11 (3) = - Cosec 23 f 111 (x) = - 2Cosecx (-Cosecx Cotx) = 2Cosec 3x Cotx

f 111 (3) = 2Cosec 33 Cot3 f (x) = f (a) + (x a) f 1 (a) + ( x a) 2 11 ( x a)3 111 f (a) + f (a) + -----------2! 3!

Log Sinx = f (3) + (x 3) f 1(3) +

( x 3) 2 11 ( x 3)3 111 f (3) + f (3) + ----------2! 3!

( x 3) 2 ( x 3)3 2 = logsin3 + (x 3) Cot3 + (-Cosec 3) + 2 Cosec 33Cot3 + --2! 3!


10 MAT11 33

Engineering Mathematics I

Dr. V. Lokesha

2011

Exercise: 25. 26. 27.

Expand Sinx is ascending powers of x 2 1 Express tan x in powers of (x 1) up to the term containing (x 1) 3

Apply Taylors Theorem to prove

h 2 h3 e x + h = e x 1 + h + + + 2! 3!

Problems on Maclaurins Expansion: 28. Expand the log (1 + x) as a power series by using Maclaurins theorem.

Solution: Here f (x) = log (1 + x), Hence f (0) = log 1 = 0 We know that

f n ( x) =

dn d n 1 1 { } + = log( 1 x ) dx n dx n 1 1 + x
n = 1,2,---------

(1) n 1 (n 1)! = , (1 + n) n
Hence f n (0) = (-1) n-1 (n 1) !

f 1 (0) = 1, f 11 (0) = -1, f 111 (0) = 3!, f 1v (0) = - 3! Substituting these values in

xn n x 2 11 f (x) = f (0) + x f (0) + f (0) + -------- + f (0) + -----------2! n!


1

log (1+x) = 0 + x . 1 +

x2 x3 x4 (-1) + 2! + - 3! + --------------4! 2! 3!

x2 x3 x4 + + --------2 3 4 This series is called Logarithmic Series.


=x10 MAT11 34

Engineering Mathematics I

Dr. V. Lokesha

2011

29. Expand tan 1 x by using Macluarins Theorem up to the term containing x 5 Solution: let y = tan 1 x, Hence y (0) = 0 1 We find that y 1 = which gives y 1 (0) = 1 1+ x2 Further y 1 (1 + x 2) = 1, Differentiating we get Y 1 . 2x + (1 + x 2) y 2 = 0 (or) (1 + x2) y 2 + 2xy 1 = 0 Hence y 2 (0) = 0 Taking n th derivative an both sides by using Leibnizs Theorem, we get n( n 1) (1 + x 2) y n + 2 + n . 2xy n +1 + . 2. y n + 2xy n 1 + n.2.y n = 0 1 .2 i.e., (1 + x 2) y n + 2 + 2 (n +1) x y n + 1 + n (n + 1) y n = 0 Substituting x = 0, we get, y n + 2 (0) = -n (n + 1) y n (0) For n = 1, we get y 3 (0) = - 2y 1 (0) = - 2 For n = 2, we get y 4 (0) = - 2 .3.y 2 (0) = 0 For n = 3, we get y 5 (0) = - 3.4.y 3 (0) = 24 Using the formula x2 x3 y 2 (0) + y 3 (0) + --------Y = y (0) + x y 1 (0) + 2! 3! x3 x5 We get tan 1 x = x + - ------------3 5 Exercise: Using Maclaurins Theorem prove the following: x2 5x 4 a) Secx = 1 + + + -------2! 4! x 3 3x 5 b) Sin 1 x = x + + + -------------6 40 x3 c) e x Cos x = 1 + x + --------------3 d) Expand e ax Cos bx by Maclaurins Theorem as far as the term containing x 3 30.

10 MAT11 35

Engineering Mathematics I

Dr. V. Lokesha

2011

Exercise : Verify Rolles Theorem for (i)


5 f ( x) = e x (sin x cos x) in , , 4 4

(ii) (iii)

f ( x) = x( x 2)e x / 2 in [0,2] f ( x) = sin 2 x e2 x


5 in , . 4 4

Exercise : Verify the Lagranges Mean Value Theorem for 1 (i) f ( x ) = x ( x 1)( x 2) in 0, 2 (ii)

f ( x) = Tan 1x in [ 0,1]

Exercise : Verify the Cauchys Mean Value Theorem for (i)


f ( x ) = x and g ( x) =

1 1 in ,1 x 4
1 in [ a, b] x

(ii) (iii)

f ( x) =

1 x
2

and g ( x) =

f ( x ) = Sin x and g ( x ) = Cos x in [ a, b]

10 MAT11 36

Engineering Mathematics I

Dr. V. Lokesha

2011

UNIT II DIFFERENTIAL CALCULUS-II


Give different types of Indeterminate Forms.
If f (x) and g (x) be two functions such that Lim f ( x ) and Lim g ( x ) both exists, then
xa xa

Lim
xa

f ( x) f ( x) Lim = xa g ( x) Lim g ( x)
xa

If Lim f ( x ) = 0 and Lim g ( x ) = 0 then


xa xa

f ( x) 0 Which do not have any definite value, such an expression is called = x a g ( x) 0 indeterminate form. The other indeterminate forms are ,0 , , 00, 0 and 1 Lim
1. State & prove L Hospitals Theorem (rule) for Indeterminate Forms. LHospital rule is applicable when the given expression is of the form Statement: Let f (x) and g (x) be two functions such that (1) Lim f ( x ) = 0 and Lim g ( x ) = 0
xa xa

0 or 0

(2) f1 (a) and g1 (a) exist and g1 (a) 0

f 1 ( x) f ( x) Lim x a = Then Lim xa g ( x) Lim g 1 ( x)


xa

1 x f ( x) 0 g Proof: Now Lim , which takes the indeterminate form . Hence applying the = Lim x a g ( x) xa 1 0 f ( x) L Hospitals theorem, we get

10 MAT11 37

Engineering Mathematics I

Dr. V. Lokesha

2011

g 1 ( x) f ( x) f ( x) [ g ( x)]2 = Lim = Lim Lim x a g ( x) x a f 1 ( x) x a f 1 ( x) g ( x) 2 [ f ( x)]


g 1 ( x) f ( x) = Lim 1 Lim x a f ( x) x a g ( x)
2

g 1 ( x)

If Lim
xa

f ( x) 0and then g ( x)

g 1 ( x) f ( x) 1 = Lim 1 Lim xa f ( x) xa g ( x)

i.e Lim
xa

f ( x) f 1 ( x) = Lim 1 g ( x) x a g ( x) f ( x) = 0 or the above theorem still holds good. g ( x)


Sinx 0 = form x 0

If Lim
xa

2. Evaluate Lim
xa

Solution: Apply LHospital rule, we get


Lim
xa

Cosx Cos 1 = = =1 1 1 1 Sinx =1 x

Lim
x a

log Sinx xa Cotx log Sinx log Sin0 log 0 Solution: Lim = form = = xa Cotx Cot 0

3. Evaluate Lim

Apply L Hospital rule = Lim


xa

Cosce 2 x 2Co sec xCo sec xCotx


10 MAT11 38

Engineering Mathematics I

Dr. V. Lokesha

2011

= Lim
xa

1 =0 2Cotx log Sinx =0 Cotx

Lim
xa

Exercise: 1 Evaluate a) Lim


x 0

tan x x
1 x

b) Lim(1 + x)
x 0

a x 1 c) Lim x x
d ) Lim
x0

xn an xa

4.

Explain - and 0 Forms:


xa xa

Solution: Suppose Lim f ( x) = 0 and Lim g ( x) = in this case


Lim f ( x) - g(x) = 0 , reduce this to
xa

0 or form 0

f ( x) 0 Let Lim[ f ( x).g ( x)] = Lim = form xa xa 1 g ( x) 0 g ( x) Or Lim[ f ( x).g ( x)] = Lim form = xa xa 1 f ( x) L Hospitals rule can be applied in either case to get the limit.

Suppose Lim f ( x) = and Lim g ( x) = in this case Lim[ f ( x).g ( x)] = form, reduce
xa xa xa

0 this or form and then apply LHospitals rule to get the limit 0

10 MAT11 39

Engineering Mathematics I
1 log(1 + x) Evaluate Lim x 0 x x2

Dr. V. Lokesha

2011

5.

1 log(1 + x) Solution: Given Lim = - form x 0 x x2 x log(1 + x 0 Required limit = Lim = form x 0 x2 0

Apply LHospital rule.


1 1 1+ x 2x

= Lim
x 0

x 1 1 = Lim 1 + x = Lim = x 0 x 0 2x 2(1 + x) 2

6.

1 Evaluate Lim Cotx x 0 x

Solution: Given limit is - form at x = 0. Hence we have


1 Cosx Required limit = Lim x0 x Sinx Sinx xCosx 0 = Lim = form x 0 xSinx 0

Apply L Hospitals rule = Lim


x 0

Cosx Cosx + xSinx xCosx + Sinx xSinx 0 = form xCosx + Sinx 0

= Lim
x 0

Apply L Hospitals rule 10 MAT11 40

Engineering Mathematics I = Lim


x 0

Dr. V. Lokesha

2011

xCosx + Sinx Cosx xSinx + Cosx

0 0 = =0 20 2

7. Evaluate Lim tan x log x


x 0

Solution: Given limit is (0 - ) form at x = 0

Required limit = Lim


x 0

log x form = Cotx

Apply L Hospitals rule = Lim


x 0

x Co sec 2 x Sin 2 x 0 form x 0

= Lim
x 0

Apply L Hospitals rule = Lim


x0

2 SinxCosx =0 1

x 8. Evaluate Lim Sec . log x x 1 2 Solution: Given limit is ( 0) form at x = 1

Required limit = Lim


x 1

log x 0 form x 0 Cos 2

Apply L Hospitals rule

= Lim
x 1

2 x = x Sin . 2 2

10 MAT11 41

Engineering Mathematics I

Dr. V. Lokesha

2011

Exercise: 2 Evaluate

x 1 a) Lim x 1 x 1 log x a x b) Lim Cot x 0 x a 1 c) Lim Secx 1 Sinx x 2


1 d) Lim a x 1 x x Explain Indeterminate Forms 0 0 , 1 , 0 , 0
g ( x)

9.

Solution: At x = a, [ f ( x)]
xa

takes the indeterminate form

(i) 0 0 if Lim f(x) = 0 and Lim g (x) = 0


xa

(ii) 1 if Lim f(x) = 1 and Lim g (x) =


xa xa

(iii) 0 if Lim f(x) = and Lim g(x) = 0 and


xa xa

(iv) 0 if Lim f(x) = 0 and Lim f (x) =


xa xa

In all these cases the following method is adopted to evaluate Let L = Lim [ f ( x)]
xa g ( x)

Lim [ f ( x)]
xa

g ( x)

so that

Log L = Lim [g(x) log f (x)] = 0


xa

Reducing this to L = ea

0 or and applying L Hospitals rule, we get Log L = a Or 0

10 MAT11 42

Engineering Mathematics I

Dr. V. Lokesha

2011

10. Evaluate Lim x Sinx


x0

Solution: let L = Lim x Sinx 00 form at x = 0


x0

Hence Log L = Lim Sinx log x 0 form


x0

LogL = Lim
x0

log x log x = Lim form x 0 Coscex 1 Sinx

Apply L Hospital rule, = Lim


x 0

Sinx. tan x 0 x form = Lim 0 x CoscexCotx x0

( )

Apply L Hospitals rule we get

= Lim
x 0

sin xSec 2 x Cosx tan x = 1


LogL = L = e = 1 1 = =0 e

L=0
11. Evaluate Lim( x)
x 1 1 1 x

Solution: let L = Lim( x)


x 1

1 x 1

is 1 form

1 0 log x form LogL = Lim x 1 1 x 0

Apply L Hospitals rule

1 = Lim x = Lim = 1 x 1 1 x 1 x
10 MAT11 43

Engineering Mathematics I

Dr. V. Lokesha

2011

Log L = -1 L = e 1 =
1 e
1 x2

tan x 12. Evaluate Lim x 0 x tan x Solution: let L = Lim x 0 x


1 x2

1 form

1 tan x LogL = Lim 2 log ( 0) form x 0 x x log tan x x LogL = Lim 2 x 0 x


Apply L Hospitals rule
xSec 2 x tan x 1 x2 = Lim 2x x0 tan x x

) 0 form
0

LogL =

xSec 2 x tan x 0 1 Lim = form 2 x 0 x3 0

Apply L Hospital rule, we get

1 Sec 2 x + 2 xSec 2 x tan x Sec 2 x = Lim 2 x 0 3x 2


= 1 tan x Lim( Sec 2 x) x 0 3 x 1 3
1 3

Log L =

L = e

10 MAT11 44

Engineering Mathematics I

Dr. V. Lokesha

2011

Exercise: 3 Evaluate the following limits.

a) Lim( Secx) Cotx


x 0

x b) Lim 2 xa a

x tan 2a

(1 + x) x e c) Lim x0 x

d) Lim(Cosax)
x 0

x2

(i ) lim

e x e x 2 log(1 + x) x 0 x sin x
1 + sin x cos x + log(1 x ) x 0 x tan 2 x

(ii ) lim

log(1 + x3 ) x 0 sin 3 x
log sin x ( x )2 2

(iii ) lim

(iv) lim
x

cosh x + log(1 x) 1 + x (v) lim x 0 x2 e2 x (1 + x) 2 x 0 x log(1 + x )

sin x sin 1 x (vi ) lim x2 x


2

(vii ) lim

Evaluate the following limits.


x (i ) lim 2 cot( x a ) xa a (ii ) lim ( cos ecx cot x )
x 0

(iii ) lim x tan x sec x 2 x 2


1 1 (v) lim 2 x 0 x x tan x
b

1 (iv) lim cot 2 x 2 x 0 x (vi ) lim [ 2 x tan x sec x ]


x

2
1

(i ) lim(cos ax) x
x 0

1 + cos x x2 (ii ) lim x 0 2

10 MAT11 45

Engineering Mathematics I
1 2 log(1 x )

Dr. V. Lokesha

2011

(iii ) lim(1 x )
x 1

sin x (iv) lim x 0 x

1 x2

(v ) lim(sin x ) tan x
x 0

(iv ) lim (1 + sin x )


x 0
2

cot x

(vii ) lim(cos x)cos ec x


x 0

(viii ) lim ( tan x )


x

tan 2 x

ax + 1 x (ix) lim( ) x ax 1
Evaluate the following limits.
b

ax + bx + cx x ( x) lim x 0 3

(i ) lim(cos ax) x
x 0

1 + cos x x2 (ii ) lim x 0 2 sin x (iv) lim x 0 x


1 x2

(iii ) lim(1 x )
x 1

1 2 log(1 x )

(v ) lim(sin x ) tan x
x 0

(iv ) lim (1 + sin x )


x 0
2

cot x

(vii ) lim(cos x)cos ec x


x 0

(viii ) lim ( tan x )


x

tan 2 x

ax + 1 x (ix) lim( ) x ax 1

ax + bx + cx x ( x) lim x 0 3

10 MAT11 46

Engineering Mathematics I

Dr. V. Lokesha

2011

Polar Curves
If we traverse in a hill section where the road is not straight, we often see caution boards hairpin bend ahead, sharp bend ahead etc. This gives an indication of the difference in the amount of bending of a road at various points which is the curvature at various points. In this chapter we discuss about the curvature, radius of curvature etc. Consider a point P in the xy-Plane. r = length of OP= radial distance = Polar angle ( r, ) Polar co-ordinates Let r = f () be the polar curve

r = x2 + y2 , = tan1 y

( x)

(1)

x = r Cos y = r Sin Relation (1) enables us to find the polar co-ordinates ( r, ) when the Cartesian co-ordinates ( x, y) are known. Expression for arc length in Cartesian form. Proof: Let P (x,y) and Q (x + x, Y + y) be two neighboring points on the graph of the function y = f (x). So that they are at length S and S + s measured from a fixed point A on the curve. Y = f (x) Q T (Tangent)

s
A

From figure,

PQ = S ,
AP = S TPR = and PR =x, RQ = y

10 MAT11 47

Engineering Mathematics I

Dr. V. Lokesha

2011

Arc PQ = S
From le PQR, we have [Chord PQ]2 = PR2 + QR2 [Chord PQ]2 = (x)2 + (y)2 ( from figure) When Q is very close to point P, the length of arc PQ is equal to the length of Chord PQ. i.e arc PQ = Chord PQ = s

(s)2 = (x)2 + (y)2 -------- (1) (x)2, we get


s y = 1+ x x
2 2

When Q P along the curve, x 0, s 0


s y Lim = 1 + Lim x 0 x x 0 x
2 2

ds dy i.e., = 1 + dx dx ds dy = 1+ dx dx
2

--------------(2)

Similarly, dividing (1) by y and taking the limit as y 0, we get

dx ds = 1+ ------------(3) dy dy

10 MAT11 48

Engineering Mathematics I

Dr. V. Lokesha

2011

ds ds & dy dx Trace a tangent to the curve at the point P, it makes an angle with the x axis. From le PRT, we have
Expressions for tan =
dy dx

Equation (2) becomes, Y

Tangent A P

T R P

T R

O
ds = 1 + tan 2 dx = Sec 2 = sec

dx ds = Sec ( Or) = Cos dx ds

and equation (3) becomes

ds = 1 + Cot 2 = Co sec 2 = Cosec dy ds dy = Cosec or = Sin dy ds

Derive an expression for arc length in parametric form. Solution: Let the equation of the curve in Parametric from be x = f (t) and y = g (t). We have, 10 MAT11 49

Engineering Mathematics I

Dr. V. Lokesha

2011

(s)2 = (x)2 + (y)2

by (t)2, we get
x y s = + t t t
2 2 2

Taking the limit as t 0 on both sides, we get


s x y Lim = Lim + Lim t 0 t t 0 t t 0 t
2 2 2

ds dx dy = + dt dt dt

(Or)

ds = dt

dx dy + dt dt

------------ (4)

Derive an expression for arc length in Cartesian form. Solution: Let P (r,) and Q (r + r, + ) be two neighboring points on the graph of the function r = f (). So that they are at lengths S and s + s from a fixed point A on the curve.

PQ = (S + s) s = s
Draw PN OQ From le OPN, Q N r + r P Tangent

r X

10 MAT11 50

Engineering Mathematics I

Dr. V. Lokesha

2011

PN = Sin OP

i.e.,

PN = Sin (or) PN = r sin r ON = Cos OP

and

i.e

ON = Cos (or) ON = r Cos r

When Q is very close to P, the length of arc PQ as equal to s, where s as the length of chord PQ. In le PQN, (PQ)2 = (PN)2 + (QN)2 but PN = r sin (Sin ) And QN = OQ ON = (r + r) r Cos = r + r r ( Cos 1)

QN = r
And (PQ)2 = (PN)2 + (QN)201 (S)2 + (r)2 + (r)2 ---------------- (5)

by ()2 we get
S r 2 = r + When Q P along the curve 0 as S 0 and r 0
2 2

10 MAT11 51

Engineering Mathematics I
S r Lim = r 2 + Lim 0 0
2 2

Dr. V. Lokesha

2011

ds dr 2 i.e = r + d d ds dr i.e., = r2 + d d
2

--------------- (6)

Similarly equation (5) by (r)2 and taking the limits as r 0 we get


S Lim = Lim r 2 + 1 r 0 r r 0 r
2 2

d ds i.e = r 2 +1 dr dr
2 2

ds = dr

d r +1 dr
2 2

--------------- (7)

Note: Angle between Tangent and Radius Vector:We have, Tan = r


d dr

i.e.,

Sin d =r Cos dr
d ds . ds dr

=r

r d Sin ds = dr Cos ds

Sin = r

d dr and Cos = ds ds

10 MAT11 52

Engineering Mathematics I

Dr. V. Lokesha

2011

ds ds and for the following curves:dy dx x 1) y = C Cos h c


Find
x Solution: y = C Cos h c Differentiating y w.r.t x. we get dy x = Sin h dx c ds dy = 1+ dx dx
2

1 + sinh 2 x
ds = Cosh dx

( c) =

Cosh 2 x

( c)

x c

Again Differentiating y w.r.t y we get


x 1 dx 1 = C Sin h c C dy

i.e

dx x = Cosech dy c
ds = 1 + Co sec h 2 x c dy

( )

2) x3 = ay2 Solution : x3 = ay2 Differentiating w.r.t y and x separately we get

10 MAT11 53

Engineering Mathematics I 3x2

Dr. V. Lokesha

2011

dx dy = 2 ay and 3 x2 = 2 ay dy dx dx 2ay dy 3 x 2 = and = dy 3 x 2 dx 2ay

i.e.,

We know that
ds dy = 1+ dx dx
2

3x 2 + 1 = 2ay
= 1+

9ay 2 x 9x3 x = 1 + 4a 2 y 2 4a 2 y 2
and
2
2

9x ds = 1+ dx 4a

dx ds 2ay = 1+ 2 = 1+ dy 3x dy ds 4a 2 y 2 = 1 + dy 9x 4
3. y = log cos x Solution . y= log cos x

2 4a 2 = 1 + 9 x

(x3 = ay2)

Differentiating w.r.t x and y separately, we get


dy 1 = ( Sin x) = - tan x dx Cosx

i.e.,

dy = - tan x dx

and 1 =

dx 1 (- Sin x) dy Cosx
10 MAT11 54

Engineering Mathematics I

Dr. V. Lokesha

2011

i.e., 1 = - tan x We have

dx dx (Or) = - Cot x dy dy

ds dy = 1+ dx dx

dx ds and = 1+ dy dy
&

ds = 1 + tan 2 x dx

ds = dy

1 + Cot 2 x

Sec 2 x
ds = (1 + Cot 2 x) 2 dy
1

ds = Sec x and dx

Find

ds for the following Curves:dt 1. x = a (Cos t + t Sin t), y = a (Sin t = t Cos t)

2. x = a Sec t , y = b tan t 3. x = a Cost + log tan t

), y = a Sin t

Solution of 1 Given x = a (Cos t + t Sin t), y = a (Sin t t Cos t ) Differentiating x & y W.r.t t, we get
dx = a (-Sin t + Sin t + t Cos t) dt dx = a t cos t dt

and

dy = a (Cos t Cos t + t Sin t) dt

dy = at Sin t dt ds dx dy = + dt dt dt
2 2

10 MAT11 55

Engineering Mathematics I =

Dr. V. Lokesha

2011

a 2 t 2 Cos 2 t + a 2 t 2 Sin 2 t

ds = at dt

Solution of 2 x = a Sec t, x = b tan t

We have

dx dy = a Sec t tan t, = b Sec2t dt dt

ds dx dy = + dt dt dt

= = =

(a2Sec2 [a2 [a2 Sec2 Sec4

t t

tan2t

b2

Sec4t) 2 b2 b2 Sec4 Sec4 t] t]


1 2 1 2 1 2

(Sec2t a2

1) + t+

Sec2

ds = [(a2 + b2) Sec4 t a2 Sec2t] dt

Solution of 3 x = a (Cos t + log tan t ), y = a Sin t 2 Differentiating x and y w.r.t t we get


dx =a dt

S int + 1 Sec 2 t . 1 , dy = a Cos t 2 2 dt tan t 2


2 2

ds dx dy = + dt dt dt

2 2 t Sec 2 2 2 2 + a Cos t = a S int + t 2 tan 2

a 2 S int Sec 2 t a 2 Sec 4 t 2 2 2 = a + 2 t t tan 4 tan 2 2

= a Cot t

10 MAT11 56

Engineering Mathematics I Find


ds dr and for the following curves d d

Dr. V. Lokesha

2011

1. r = a (1 Cos ) 2 .r2 = a2 Cos 2 3. r = a eCot Solution of 1 r = a (1 Cos ) Differentiating r w.r.t we get


dr = a Sin d

Hence
2 2 ds dr = r 2 + d d 1 1

= {a2 (1 Cos )2 + a2 Sin2 } 2


1

= {a2 (1 2 Cos + Cos2) + a2 Sin2 } 2


1

= {a2 2a2 Cos + a2} 2


1

= {2a2 2a2 Cos } 2


1

= a{ 2(1 Cos )} 2 = a {2 (2 Sin2


ds = 2 a Sin 2 d
2 2 ds d And = 1 + r 2 dr dr 1

)}

1 2

a (1 Cos ) = 1 + a 2 Sin 2
2 2

1 2

10 MAT11 57

Engineering Mathematics I

Dr. V. Lokesha
1

2011

Sin 2 + (1 Cos ) 2 2 = Sin 2

{2(1 Cos )}
Sin

1 2

2Sin 2Sin 2 2 = = Sin 2 Sin Cos 2 2

1 ds = dr Cos
Solution of 2

r2 = a2 Cos 2 Differentiating W.r.t we get 2r r


dr = -a2 Sin 2 . 2 d dr = -a2 Sin 2 d

a 2 Sin2 dr = r d
Hence
2 2 ds 2 dr = r + d d 1

a 2 Sin 2 = r 2 + r

a 4 Sin 2 2 2 = r 2 + r2 a 4 Sin 2 2 = a 2 Cos 2 + 2 a Cos 2 a 4 Cos 2 2 + a 4 Sin 2 2 2 = a 2 Cos 2


10 MAT11 58
1 1

Engineering Mathematics I

Dr. V. Lokesha

2011

a4 2 a = 2 = Cos 2 a Cos 2 a 2 a2 ds Or = 2 = (r2 = a2 Cos 2) r r d


4
2 ds d And = 1 + r 2 dr dr 1 2

2 r = 1 + r 2 2 a Sin 2

r4 = 1 + 4 2 a Sin 2

a 4 Sin 2 2 + a 4 Cos 2 2 = a 4 sin 2 2


1

2 a4 1 = 4 = Cosec 2 = 2 Sin 2 a sin 2


Solution 3 r = aeCot , here is constant Differentiating w.r.t we get
dr = a eCot . Cot d

Hence
2 2 ds 2 dr = r + d d 1

{a2

e2Cot

a2

e2Cot
1

cot2

1 2

=a =a

eCot eCot

{1 +

Cot2} 2
1

{Cosce2} 2 10 MAT11 59

Engineering Mathematics I
ds = a eCot Cosce d
2 ds 2 d and = 1 + r dr dr 1 2

Dr. V. Lokesha

2011

1 = 1 + a 2 e 2Cot 2 2CotCot 2 a e

= {1 + tan2 }
1

1 2

= {Sec2 } 2 = Sec Exercises:

ds ds and to the following curves. dr d 1. rn = an Cos n

Find

2. r (1 + Cos ) = a 3. r = a Note: We have Sin = r Cos =


dr ds
1 2

d and ds

dr = Cos = (1 Sin2) ds
2 1 2

p = 1 2 Since P = r Sin . r
dr = ds

r 2 p2 r

10 MAT11 60

Engineering Mathematics I

Dr. V. Lokesha

2011

ds = dr

r r p2
2

r+r

r O P R

P(x,y) OR = P X Sin =
OR p = OP r

P = r Sin
d Prove that with usual notations tan = r dr

Let P (r, ) be any point on the curve r = f ()


X O P = and OP = r

Let PL be the tangent to the curve at P subtending an angle with the positive direction of the initial line (x axis) and be the angle between the radius vector OP and the tangent PL. That is O P L = From the figure we have 10 MAT11 61

Engineering Mathematics I

Dr. V. Lokesha

2011

= +
(Recall from geometry that an exterior angle is equal to the sum of the interior opposite angles)
tan = tan ( + )

or tan

tan + tan 1 - tan tan

(1)

Let (x, y) be the Cartesian coordinates of P so that we have, X = r cos , y = r sin Since r is a function of , we can as well regard these as parametric equations in terms of . We also know from the geometrical meaning of the derivative that
tan = dy = slope of the tangent PL dx dy d dx d

ie., tan =

since x and y are function of

d (r sin ) r cos + r sin dr = where r = ie., tan = d d d (r cos ) - r sin + r cos d


Dividing both the numerator and denominator by r cos we have,

r cos r sin + tan = r cos r cos r sin r cos + r cos r cos


Or

r + tan r tan = r 1 - . tan r


r r d = or tan = r r dr dr d

(2)

Comparing equations (1) and (2) we get


tan =

10 MAT11 62

Engineering Mathematics I

Dr. V. Lokesha

2011

Prove with usual notations


2

1 dr 1 1 = 2+ 4 2 r d r p

or

1 1 du = u 2 + where u = 2 r p d

Proof :
Let O be the pole and OL be the initial line. Let P (r, ) be any point on the curve and hence we have OP = r and L O P = Draw ON = p (say) perpendicular from the pole on the tangent at P and let be the angle made by the radius vector with the tangent. From the figure O N P = 90 L O P = Now from the right angled triangle ONP
sin = ON OP P r or p = r sin

ie., sin =

we have p = r sin and cot =


1 dr r d

(1) (2)

Squaring equation (1) and taking the reciprocal we get, 10 MAT11 63

Engineering Mathematics I
1 1 1 = 2 . 2 p r sin 2 1 1 = 2 cosec 2 2 p r

Dr. V. Lokesha

2011

ie.,

Or

1 1 = 1 + cot 2 2 2 p r

Now using (2) we get,

1 1 = 2 2 p r
Or

1 1 + 2 r

dr d

1 1 1 = 2 + 4 2 r r p
1 =u r

dr d

(3)

Further, let

Differentiating w.r.t. we get,

1 dr 1 dr du du 2 = 4 = , by squaring r d r d d d
Thus (3) now becomes

1 du = u2 + 2 p d

(4)

1. Find the angle of intersection of the curves:


r = a (1 + cos ) & r = b (1 - cos )

Solution : r = a (1 + cos )
log r = log a + log (1 + cos )
Differentiating these w.r.t. we get
1 dr - sin = 0 + r d 1 + cos

: :

r = b(1 cos ) log r = log b + log (1 cos )

: :

1 dr sin = 0 + r d 1 cos

cot 1 =

2 sin (/2 ) cos (/2 ) 2 cos 2 (/2 )

cot 2 =

2 sin (/2 ) cos (/2 ) 2 sin 2 (/2 )

ie., cot 1 = - tan ( /2 ) = cot (/2 + /2 )


1 = /2 + /2

cot 2 = cot ( /2 )

2 = /2

10 MAT11 64

Engineering Mathematics I

Dr. V. Lokesha

2011

angle of intersection = 1 - 2 = /2 + /2 - /2 = /2
Hence the curves intersect orthogonally. 2. S.T. the curves
r = a (1 + sin ) & r = a (1 - sin )

cut each other orthogonally

Solution :

log r = log a + log (1 + sin ) : log r = log a + log (1 sin ) Differentiating these w.r.t we get
1 dr cos = r d 1 + sin 1 dr - cos = r d 1 sin cot 2 = - cos 1 sin

: :

ie.,

cot 1 =

cos 1 + sin

We have tan 1 =

1 + sin 1 - sin and tan 2 = cos - cos

1 - sin 2 cos 2 = tan 1 . tan 2 = = -1 - cos 2 cos 2

Hence the curves intersect orthogonally.

3.

Find the angle of intersection of the curves: r = sin + cos , r = 2 sin Solution : log r = log (sin + cos ) : r = 2 sin log r = log 2 + log (sin )
log r = log (sin + cos ) :

Differentiating these w.r.t we get


1 dr cos - sin = r d sin + cos

1 dr cos = r d sin

ie.,

cot 1 =

cos (1 - tan ) cos (1 + tan )

cot 2 = cot 2 =

ie., cot 1 = cot (/4 + ) 1 = /4 +


1 - 2 = /4 + - = /4

The angle of intersection is / 4

10 MAT11 65

Engineering Mathematics I

Dr. V. Lokesha

2011

4. Find the angle of the curves: r = a log and r = a/ log Solution : r = a log : r = a/ log : log r = log a - log (log )
log r = log a + log (log )

Differentiating these w.r.t q, we get,

1 dr 1 = r d log .
ie.,

: :

1 dr 1 = r d log . cot 2 = 1 log

cot 1 =

1 log

Note : We can not find 1 and 2 explicitly.


tan 1 = log

tan 2 = log tan 2 = - log

Now consider, tan 1 = log : Now consider, tan (1 - 2 ) =

2 log 2 log = 1 + tan 1 tan 2 1 ( log )2

......(1)

We have to find by solving the given pair of equations :

r = a log and r = a/log


Equating the R.H.S we have a log = ie., (log = 1 )2 = 1 or log = e Substituting = e in (1) we get
tan (1 2 ) = 2e 1 - e2

a log

( log e = 1)

2e angle of intersection 1 - 2 = tan -1 = 2 tan -1 e 2 1- e

5. Find the angle of intersection of the curves: r = a (1 cos ) and r = 2a cos Solution : r = a (1 cos ) Taking logarithms we have, Log r = log a + log (1 cos ) : log r = log 2a + log (cos ) 10 MAT11 66 Differentiating these w.r.t , we get,

r = 2a cos

Engineering Mathematics I
sin 1 dr = r d 1 - cos

Dr. V. Lokesha :
1 dr - sin = r d cos

2011

ie., cot 1 =

2 sin (/2 ) cos (/2 ) : cot 2 = - tan 2 sin 2 (/2 )

ie., cot 1 = cot ( /2 )


1 = /2

: :

cot 2 = cot (/2 + ) 2 = /2 +

1 2 = /2 - /2 -

= / 2 + 2

(1)

Now consider r = a (1 - cos ) = 2a cos Or


3 cos = 1 or = cos -1 (1/3)

Substituting this value in (1) we get, The angle of intersection / 2 + 1/2 . cos -1 (1/3) 6. Find the angle of intersection of the curves :

r = a and r = a /
Solution : r = a : :

r=a/
log r = log a - log

log r = log a + log

Differentiating these w.r.t , we get,


1 1 dr = r d

: : :

1 1 dr = r d cot 2 = 1

ie., or

cot 1 =

tan 1 =

tan 2 = -

Also by equating the R.H.S of the given equations we have


a = a/ or 2 = 1 = 1

When = 1, tan 1 = 1, tan 2 = - 1 and When = - 1, tan 1 = - 1, tan 2 = 1 .


tan 1 . tan 2 = - 1 1 - 2 = /2

The curves intersect at right angles.

10 MAT11 67

Engineering Mathematics I

Dr. V. Lokesha

2011

7. Find the pedal equation of the curve: r (1 - cos ) = 2a Solution : r (1 - cos ) = 2a


log r + log (1 - cos ) = log 2a

Differentiating w.r.t , we get


- sin 1 dr sin 1 dr = 0 or = + r d 1 - cos r d 1 - cos

cot =

- 2 sin (/2 ) cos (/2 ) = - cot (/2 ) 2 sin 2 (/2 )

ie., cot = cot (- /2 ) = - ( /2 ) Consider p = r sin


p = r sin (- /2 ) or p = - r sin ( /2 )

Now we have, r (1 - cos ) = 2a


p = - r sin ( /2 )

(1) (2)

We have to eliminate from (1) and (2) (1) can be put in form r . 2 sin 2 ( / 2 ) = 2a ie., r sin 2 (/2 ) = a But p/-r = sin (/2), from (2)
p2 2 r r 2 = a or p = ar

Thus p2 = ar is the required pedal equation. 8. Find the pedal equation of the curve: r2 = a2 sec 2 Solution : r2 = a2 sec 2

2 log r = 2 log a + log (sec 2)


Differentiating w.r.t , we get,
2 dr 2 sec 2 tan 2 1 dr = ie., = tan 2 r d sec 2 r d

ie., cot = cot ( /2 - 2 ) = /2 - 2 10 MAT11 68

Engineering Mathematics I Consider p = r sin p = r sin ( /2 - 2 ) ie., p = r cos 2 Now we have, r 2 = a 2 sec 2

Dr. V. Lokesha

2011

(1) (2)

p = r cos 2
From (2) p/r = cos 2 or r/p = sec 2 Substituting in (1) we get,
r 2 = a 2 (r/p )

or pr = a2

Thus pr = a2 is the required pedal equation. 9. Find the pedal equation of the curve: rn = an cos n Solution : rn = an cos n
n log r = n log a + log (cos n )

Differentiating w.r.t q we get


1 dr n dr - n sin n = - tan n = ie., r d cos n r d
cot = cot ( /2 + n ) = /2 + n

Consider p = r sin
p = r sin ( /2 = n ) ie., p = r cos n

Now we have, r n = a n cos n

(1) (2)

p = r cos n
(1) as a consequenc e of (2) is r n = a n (p/r )

Thus rn + 1 = pan is the required pedal equation. 10. Find the pedal equation of the curve: rm = am (cos m + sin m) Solution : rm = am (cos m + sin m) Differentiating w.r.t , we get,
m dr - m sin m + m cos m = cos m + sin m r d

ie.,

cos m (1 - tan m ) 1 dr cos m - sin m = = r d cos m + sin m cos m (1 + tan m )

cot = cot ( /4 + m ) = /4 + m

10 MAT11 69

Engineering Mathematics I Consider p = r sin


p = r sin ( /4 + m )

Dr. V. Lokesha

2011

ie., p = r [sin ( /4 ) cos m + cos (/4 ) sin m ] ie., p =


r (cos m + sin m ) 2

(we have used the formula of sin (A + B) and also the values sin (/4 ) = cos (/4 ) = 1/ 2 Now we have, r m = a m (cos m + sin m )
p= r (cos m + sin m ) 2

(1) (2)

Using (2) in (1) we get,


rm = am . p 2 or r m +1 = 2 a m p r

Thus r m + 1 = 2 a m p is the required pedal equation. 11. Establish the pedal equation of the curve:

r n = a n sin n + b n cos n in the form p 2 a 2n + b 2 n = r 2 n + 2

Solution : We have r n = a n sin n + b n cos n


n log r = log a n sin n + b n cos n

Differentiating w.r.t we get


n dr na n cos n - nb n sin n = n r d a sin n + b n cos n a n cos n - b n sin n Dividing by n, cot = n a sin n + b n cos n

Consider p = r sin Since cannot be found, squaring and taking the reciprocal we get,
1 1 1 1 = 2 cosec 2 or 2 = 2 1 + cot 2 2 p r p r

1 1 2= 2 p r

2 a n cos n - b n sin n 1 + n 2 n a sin n + b cos n

( (

) )

10 MAT11 70

Engineering Mathematics I
1 1 ie., 2 = 2 p r
1 1 ie., 2 = 2 p r

Dr. V. Lokesha

2011

2 n n n n a sin n + b cos n + a cos n - b sin n 2 a n sin n + b n cos n

) (

)
2

2n 2 2 2n 2 2 a sin n + cos n + b cos n + sin n 2 n n a sin n + b cos n

(product terms cancels out in the numerator)


ie., 1 1 a 2 n + b2 n = . p 2 r 2 a n sin n + b n cos n

or

1 1 a 2 n + b 2n . , by using the given equation. = 2 p2 r 2 rn

p2 a 2 n + b2 n

( ) )= r

2 n+2

is the required pedal equation.

12. Define Curvature and Radius of curvature Solution: A Curve Cuts at every point on it. Which is determined by the tangent drawn.

y = f (x)

Tangent

P (x,y)

X O Let P be a point on the curve y= f (x) at the length s from a fixed point A on it. Let the tangent at P makes are angle with positive direction of x axis. As the point P moves along curve, both s and vary. d as called the Curvature of the curve at P. The rate of change w.r.t s, i.e., ds The reciprocal of the Curvature at P is called the radius of curvature at P and is denoted by .

ds 1 = d d ds
10 MAT11 71

Engineering Mathematics I
1 d ds (or) = d ds 1 Also denoted = K K is read it as Kappa. =

Dr. V. Lokesha

2011

K =

d ds

13. Derive an expression for radius of curvature in Cartesian form. Solution :(1) Cartesian Form: Y = f (x) A S P T dy

O X

P dx R

Let y = f (x) be the curve in Cartesian form. dy We know that, tan = (From Figure) ----------- (1) dx Where is the angle made by the tangent at P with x axis. Differentiating (1) W.r.t x, we get

d2y d = dx dx 2 d2y d i.e., = Sec2 2 dx dx d = (1 + tan2 ) dx d ds = (1 + tan2 ) . dx dx


Sec2 .
2 2 1 dy 2 = (1 + tan ) 1 + dx 2 2 2 dy 1 dy = 1 + . . 1 + dx dx 1 1

from eqn (1)

10 MAT11 72

Engineering Mathematics I
2 2 dy + 1 dx 3

Dr. V. Lokesha

2011

dy 2 2 1 + dx = 2 d y dx 2 = ( 1+ y1 2 )3/2 (1) y2 d2y dy Where y1 = , y2 = dx dx 2 This is the formula for Radius of Curvature in Cartesian Form. 14. Show that the Curvature of a Circle at any point on it, is a Constant
Tangent O r PP P

Solution: Consider a Circle of radius r. Let A be a fixed point and P be a given point on the circle such that arc AP = S. Let the angle between the tangent to the Circle at A and P be . Then clearly AOP = .

AP = r i.e., S = r This is the intrinsic equation of the circle. Differentiating w.r.t S we get
d d 1 Or K = = ds ds r 1 K = which is Constant r

1=r

Hence the Curvature of the Circle at any point on it is constant. 10 MAT11 73

Engineering Mathematics I

Dr. V. Lokesha

2011

15. Derive an expression for radius of curvature in parametric form. Solution: We have = (1 + y 12 )3/2 y2 Let x =f (t), y = g (t) be the curve in Parametric Form. dy Y dy dt Then y1 = = = dx dx dt X d2y d dy d Y d Y Y2 = = = dx dx dx dx 2 X dt X d Y 1 d Y 1 = . . dt X dx dt X X dt xy yx 1 = (x )2 ( X )
x Substituting Y1 and Y2 in equation 1.

dt dx

Y2 =

X y yx
3

(1 + Y ) =
2 1

y2

, we get
3

-------------------- (2) xy yx Equation (2) is called the Radius of Curvature in Parametric Form.

[(x) + ( y ) ] =
2 2

1+ Y 2 x = xy yx (x )3
3

( ( )

10 MAT11 74

Engineering Mathematics I

Dr. V. Lokesha

2011

16. Derive an expression for radius of curvature in polar form. Solution: Let r = f () be the curve in the Polar Form. We know that, Angle between the tangent and radius vector, Tan = r
d dr

= r.

1 dr d

i.e., tan =

Differentiate w.r.t we get

(dr d )

dr dr d 2r . r 2 d Sec2 . = d d 2 d d dr d
2 dr d 2r 2 d 1 d r d = 2 2 d Sec dr d

2 dr d 2r 2 r 1 d d = 2 2 1 + tan dr d 2 dr d 2r r 1 d 2 d = 2 r2 dr 1+ 2 dr d d 2 d 2r dr r 2 d d d = 2 d dr 2 +r d

) (

From figure = + Differentiating w.r.t , we get 10 MAT11 75

Engineering Mathematics I
d d =1+ d d

Dr. V. Lokesha

2011

d 2r dr r d d d 2 =1+ 2 d dr 2 +r d d 2r dr r 2 + 2 r d d 2 d = 2 d dr 2 +r d
2 2 ds dr Also we know that = r 2 + d d 1 2

Now, =

ds ds d = . d d d
2 1 2

dr 2 +r 2 dr d = r 2 + . 2 d dr d 2r r 2 + 2 r d 2 d
+ r12 ) 2 = 2 ----------- (3) r + 2r12 rr2
2 3

(r

d 2r dr Where r1 = , r2 = d d 2
Equation (3) as called the radius of curvature in Polar form. 17. Derive an expression for radius of curvature in pedal form. Solution: Let p = r Sin be the curve in Polar Form. We have p = r Sin Differentiating p W.r.t r, we get
dp d = Sin + r Cos dr dr

10 MAT11 76

Engineering Mathematics I

Dr. V. Lokesha

2011

But Sin = r

d dr , Cos = ds ds

d dr d dp =r +r dr ds ds dr d d dr + . ds dr ds d d +r ds ds d ( + ) ds d ds d dp =r dr ds

=r

=r

=r

=r

1 dp = r. dr

1 d = ds

= r

dr dp

--------------- (4)

Equation (4) is called Radius of Curvature of the Curve in Polar Form. 18. Find the radius of curvature at (x, y) for the curve ay2 = x3. Solution: Given ay2 = x3 -------- (1) is in Cartesian form. We have, Radius of curvature in Cartesian form.

(1 + y ) =
2 1

y2

------------ (2)

Differentiating (1) w.r.t x we get,

10 MAT11 77

Engineering Mathematics I

Dr. V. Lokesha

2011

a 2y

dy dy 3 x2 = 3x 2 = y1 = = dx dx 2 ay

3x 2 x3 2a a
1 2

3 x 2 a

y1 =

3 x 2 a

Differentiating y1 w.r.t x we get

y2 =

3 1 3 d2y = = 2 dx 2 a 2 x 4 a x
x (4a + 9 x ) 6a

Substitute Y1 and Y2 in (2), we get.

x 19. Find the radius of curvature at (x,y) for the curve y = c log Sec c x Solution: Given y = c log Sec -------- (1) c Differentiating (1) w.r.t x, we get

1 x x .Sec tan = c tan x c c Sec c Differentiating y1 W.r.t x we get y1 = c.


x 1 1 x y 2 = Sec 2 = Sec 2 c c c c

x 1 . = tan c c

x c

(1 + y ) Substitute y1 and y2 in =
2 1

[1 + tan (x c )] = [Sec (x c )] = 1 1 Sec (x ) Sec (x ) c c c c


2 3 2 2 3 2 2 2

y2

= cSec x

( c)
10 MAT11 78

Engineering Mathematics I

Dr. V. Lokesha

2011

20. Find the radius of curvature at the point t on the curve x = a (t + Sint), y = a (1 Cost). Solution: Given Curves are in Parametric Form

((x) Radius of Curvature, =


x=

+ (y) 2 ------ (1) xy yx Differentiating the given Curves W.r.t t, we get


2 2 3

dx dy = a (1 + Cost) y = = a Sint dt dt Differentiating W.r.t t we get

x = - a Sint,

y = a Cost

Substitute x , y , y and x in (1), we get


+ a 2 S int 2 = a(1 + Cost )aCost aS int( aS int)
2 2

(a (1 + Cost )

a3{ 1 + 2Cost + Cos 2 t + Sin 2 t } 2 = a 2 {Cost + Cos 2 t + Sin 2 t}


3

a 2(1 + Cost ) = (1 + Cost )

}=

a 2.2Cos 2 t 2Cos t
2

} 2

8a.Cos 3 t 2Cos t
2

= 4aCos t

2 21. Find the Radius of Curvature to x = a Cost + log tan t

( 2 )}, y = a Sint at t.

Solution: Here x = a Cost + log tan t

( 2 )}, y = a Sin t

dx 1 1 2 t = a S int + .Sec . 2 dt 2 tan t 2 1 } =a { -sint + 2 sin t / 2 cos t / 2 1 =a { -sint + } 2 sin t / 2 = a { (1-sin 2t) / sint }
dx = a Cos2t / sint dt

10 MAT11 79

Engineering Mathematics I
and dy = aCost dx aCost = tant a cos t cos t / sin t

Dr. V. Lokesha

2011

dy dy / dt = = dx dx / dt

dy = tan t dx Differentiating W.r.t x we get dt d2y 1 1 = Sec 2 t = Sec 2 t = Sec 2 t. 2 2 dx dx dx aCos t S int dt d2y 1 = = Sec 4 tS int 2 a dx

(1 + y12 ) d2y dy Substitute & in = dx y2 dx 2


i.e.,

, we get

1 Sec 4tS int a = a Cot t.

(1 + tan t ) =
2

aSec3t Cost =a = aCott 4 Sec tS int S int

22. Find the Radius of Curvature to Solution: Given

a a x + y = a at , 4 4

x+ y= a

-------------- (1)

Differentiating (1) w.r.t to x we get

1 2 x

1 dy =0 2 y dx
y dy = = dx x

i.e., y1 =

a x x

) (From (1))

y1 = 1

a x

----------- (2)

10 MAT11 80

Engineering Mathematics I Also y 2 =

Dr. V. Lokesha

2011

d2y a 1 3 = a x 2 = 3 2 dx 2 2x 2

-------------- (3)

a a At the given point , 4 4 a a 4 Then y1 = 1 = - 1 & y2 = 1 = 3 a a 2a 2 2 4

( )

Substitute y1 and y2 in

(1 + y ) =
2 1

(1 + (1) ) =
2

y2

2 2 a 2 2a a = = = 4 4 2

a 2
23. Show that for the Cardioids r = a (1 + Cos), 2 / r 2 is a constant

Solution: r = a (1 + Cos)
dr = - a Sin d

We have,
1 1 1 dr = 2 + 4 , is Pedal Equation. 2 P r r d
2

1 1 + 4 a2Sin2 2 r r

r 2 + a 2 Sin 2 a 2 (1 + Cos ) 2 + a 2 Sin 2 = r4 r4

2a 2 (1 + Cos ) = r4

10 MAT11 81

Engineering Mathematics I

Dr. V. Lokesha ( r = a (1 + Cos)


r = 1 + Cos a

2011

1 2a 2 = P2 r4

r a

1 2a = 3 2 r P

P2 =

r3 2a

Differentiating w.r.t P we get 1 2 dr 2P = 3r dp 2a

dr 4ap = dp 3r 2
Now,

=r

dr 4ap = dp 3r

And

2
r2

1 16a 2 p 2 16a 2 r 3 8a . = = r 9r 2 9r 3 2a 9
1 1 1 r2 = + P 2 a 2 b 2 a 2b 2

24.

Find the Radius of Curvature of the Curve

Solution: Given

1 1 1 r2 = + P 2 a 2 b 2 a 2b 2

Differentiating w.r.t to P, we get

dr 2 1 = 2 2 2r 3 dp P a b dr a 2 b 2 = 3 dp p r

dr a 2b 2 a 2b 2 = r. = r. 3 = 3 dp pr p

10 MAT11 82

Engineering Mathematics I

Dr. V. Lokesha

2011

25. Find the Radius of Curvature at (r,) on r = Solution: Given r =


a

Differentiating w.r.t to we get


dr a a 1 r = = . = d 2 dr r = d 1 1 1 dr We have 2 = 2 + 4 P r r d 1 1 r2 1 1 = 2 + 4 2 = 2 + 2 2 r r r r
2

1 1 1 1 2 + 1 = 1 + = 2 P2 r 2 2 r 2 P =
P=

2 +1
r

a2 a2 + r 2 + 1 r2 Differentiating above result w.r.t toP we get


a 2 + r 2 .a 1=
2 2

2 +1

r. a

a.r

dr 1 dr /r r.a 2 dp dp / a2 + r2 2 2 2 a +r

2 r 2a 2 a +r = a +r .a a2 + r 2
a2 + r 2 = (a 2 + r 2 ). a r 2 a dr dp a2 + r 2

dr dp

10 MAT11 83

Engineering Mathematics I

Dr. V. Lokesha

2011

(a (a

+ r 2 ) a2 + r 2 = a3 + r2 ) a3
3 2

dr dp

dr r. a 2 + r 2 Thus, = r. = dp a3 3 r = 3 (a 2 + r 2 ) 2 a

dr dp

Exercises:

(1) Find the Radius of the Curvature at the point (s, ) on S = a log tan + 4 2

(2) Find the Radius of the Curvature of xy = C2 at (x,y) (3) Find the Radius of the Curvature of xy3 = a4 at (a,a) (4) Find the Radius of Curvature at the point on x = C Sin 2 (1 + Cos 2), y = C Cos 2 (1 Cos 2) (5) If 1 and 2 are the radii of curvature at the extremities of any chord of the cardiode 16a 2 2 2 r = a (1 + Cos) which posses through the Pole prove that 1 + 2 = 9

10 MAT11 84

Engineering Mathematics I

Dr. V. Lokesha

2011

DIFFERENTIAL CALCULUS III

PARTIAL DIFFERENTIATION Introduction :Partial differential equations abound in all branches of science and engineering and many areas of business. The number of applications is endless. Partial derivatives have many important uses in math and science. We shall see that a partial derivative is not much more or less than a particular sort of directional derivative. The only trick is to have a reliable way of specifying directions ... so most of this note is concerned with formalizing the idea of direction So far, we had been dealing with functions of a single independent variable. We will now consider functions which depend on more than one independent variable; Such functions are called functions of several variables.

Geometrical Meaning
Suppose the graph of z = f (x,y) is the surface shown. Consider the partial derivative of f with respect to x at a point (x0, y0). Holding y constant and varying x, we trace out a curve that is the intersection of the surface with the vertical plane y = y0. The partial derivative fx(x0,y0). measures the change in z per unit increase in x along this curve. That is, fx(x0, y0) is just the slope of the curve at (x0, y0). The geometrical interpretation of fy(x0, y0). is analogous.

10 MAT11 85

Engineering Mathematics I

Dr. V. Lokesha

2011

Real-World Applications: Rates of Change: In the Java applet we saw how the concept of partial derivative could be applied geometrically to find the slope of the surface in the x and y directions. In the following two examples we present partial derivatives as rates of change. Specifically we explore an application to a temperature function ( this example does have a geometric aspect in terms of the physical model itself) and a second application to electrical circuits, where no geometry is involved. I. Temperature on a Metal Plate The screen capture below shows a current website illustrating thermal flow for chemical engineering. Our first application will deal with a similar flat plate where temperature varies with position. * The example following the picture below is taken from the current text in SM221,223: Multivariable Calculus by James Stewart.

10 MAT11 86

Engineering Mathematics I

Dr. V. Lokesha

2011

Suppose we have a flat metal plate where the temperature at a point (x,y) varies according to position. In particular, let the temperature at a point (x,y) be given by,

T ( x, y ) = 60 /1 + x 2 + y 2
where T is measured in oC and x and y in meters. Question: what is the rate of change of temperature with respect to distance at the point (2,1) in (a) the x-direction? and (b) in the y-direction ? Let's take (a) first. What is the rate of change of temperature with respect to distance at the point (2,1) in (a) the x-direction? What observations and translations can we make here? Rate of change of temperature indicates that we will be computing a type of derivative. Since the temperature function is defined on two variables we will be computing a partial derivative. Since the question asks for the rate of change in the x-direction, we will be holding y constant. Thus, our question now becomes: What is dT dxat the point (2,1)?
T ( x, y ) = 60 /1 + x 2 + y 2 = 60(1 + x 2 + y 2 ) 1 T T x x = 60(2 x)(1 + x 2 + y 2 ) 2 (2,1) = 60(4)(1 + 4 + 1) 2 = 20 3

Conclusion : The rate of change of temperature in the x-direction at (2,1) is 20 3 degrees per meter; note this means that the temperature is decreasing !

10 MAT11 87

Engineering Mathematics I

Dr. V. Lokesha

2011

Part (b): The rate of change of temperature in the y-direction at (2,1) is computed in a similar manner.
T ( x, y ) = 60 /1 + x 2 + y 2 = 60(1 + x 2 + y 2 ) 1 T T x x = 60(2 y )(1 + x 2 + y 2 ) 2 (2,1) = 60(2)(1 + 4 + 1) 2 = 10 3

Conclusion : The rate of change of temperature in the y-direction at (2,1) is 10 degrees


3

per meter; note this means that the temperature is decreasing !

II. Electrical Circuits: Changes in Current


The following is adapted from an example in a former text for SM221,223 Multivariable Calculus by Bradley and Smith. * In an electrical circuit with electromotive force (EMF) of E volts and resistance R ohms, the current, I, is I=E/R amperes. Question: (a) At the instant when E=120 and R=15 , what is the rate of change of current with respect to voltage. (b) What is the rate of change of current with respect to resistance?

(a) Even though no geometry is involved in this example, the rate of change questions can be answered with partial derivatives. we first note that I is a function of E and R, namely, I(E,R) = ER-1
10 MAT11 88

Engineering Mathematics I

Dr. V. Lokesha

2011

The rate of change of current with respect to voltage = the partial derivative of I with respect to voltage, holding resistance constant is
I E = R 1

when E=120 and R=15 , we have I E = 15 1 0.0667 verbal conclusion : If the resistance is fixed at 15 ohms, the current is increasing with respect to voltage at the rate of 0.0667 amperes per volt when the EMF is 120 volts. Part (b): What is the rate of change of current with respect to Using similar observations to part (a) we conclude: The partial derivative of I with respect to resistance, holding voltage constant = I E = ER 1 when E=120 and R=15 , we have I E (120,15) = 120(15)1 0.5333 Conclusion : If the EMF is fixed at 120 volts, the current is decreasing with respect to resistance at the rate of 0.5333 amperes per ohm when the resistance is 15 ohms. resistance?

Key Words :Then the partial derivative of z w.r.t x is given by


zx = z f ( x + x, y ) f ( x, y ) = lim x x x0

The partial derivative of z w.r.t y is given by


zy = z f ( x, y + y ) f ( x, y ) = lim y y 0 y

10 MAT11 89

Engineering Mathematics I 1. If u = e ax -by sin (ax + by ) show that b

Dr. V. Lokesha

2011

u u -a = 2 ab u y x

Solution : u = e ax -by sin (ax + by )


u = e ax - by cos (ax + by ) . a + a.e ax -by sin (ax + by ) x u = a e ax - by cos (ax + by ) + au x u = e ax - by cos (ax + by ) . b + (- b ) e ax -by sin (ax + by ) x

ie.,

(1)

Also

ie.,

u = b e ax - by cos (ax + by ) bu y u u =a by using (1) and (2) becomes x y

(2)

Now b

= abe ax by cos (ax + by ) + abu - abe ax -by cos (ax + by ) + abu

= 2 abu Thus b 2.

u u - a = 2 abu x y

If u = eax + by f (ax - by ), prove that

u u =a = 2abu x y

Solution : u = e ax + by f (ax - by ) , by data


u = e ax + by . f (ax - by ) a + ae ax + by f (ax - by ) x

Or Next, Or

u = a e ax + by . f (ax - by ) + a u x

(1)

u = e ax + by f (ax - by ) . (- b ) + b e ax + by f (ax - by ) y u = b e ax + by f (ax - by ) + ba y u u + a y x


10 MAT11 90 (2)

Now consider L.H.S = b

Engineering Mathematics I

Dr. V. Lokesha

2011

= b ae ax + by f (ax - by ) + au + a - be ax + by f (ax - by ) + bu = ab e ax + by f (ax - by ) + abu - ab e ax + by f (ax - by ) + abu

} {

= 2abu = R.H.S Thus b 3.

u u +a = 2a bu x y

2u 2u 2u If u = log x 2 + y 2 + z 2 , show that x 2 + y 2 + z 2 =1 x 2 + y 2 + z 2

Solution : By data u = log x 2 + y 2 + z 2 = log (x 2 + y 2 + z 2 )


1 2

The given u is a symmetric function of x, y, z, (It is enough if we compute only one of the required partial derivative)
1 u 1 x . 2x = 2 = . 2 2 2 x 2 x + y + z x + y2 + z 2

2 u u x = = 2 2 2 2 x x x x + y + z x

(x ie.,

+ y2 + z2 1 - x . 2 x

(x

+ y2 + z2

(x

y2 + z2 x2
2

+ y2 + z2

2u y2 + z2 x2 = 2 x 2 x2 + y2 + z2

(1)

Similarly

2u z2 + x 2 y2 = 2 y 2 x2 + y2 + z2

(2)

2u x 2 + y2 z 2 = 2 z 2 x2 + y2 + z2

(3)

Adding (1), (2) and (3) we get,

x2 + y2 + z2 2u 2u 2u 1 + + = = 2 2 2 2 2 x y z x + y2 + z 2 x2 + y2 + z2

2u 2u 2u Thus x 2 + y 2 + z 2 x 2 + y 2 + z 2 = 1

10 MAT11 91

Engineering Mathematics I 4. If u = log (tan x + tan y + tan z), show that, sin 2x ux + sin 2y uy + sin 2z uz = 2

Dr. V. Lokesha

2011

Solution : u = log (tan x + tan y + tan z) is a symmetric function.


ux = sec 2 x tan x + tan y + tan z

sin 2 x u x =
Or

(2 sin x cos x ) sec2 . x


tan x + tan y + tan z
sin 2 x u x = 2 tan x tan x + tan y + tan z 2 tan y tan x + tan y + tan z 2 tan z tan x + tan y + tan z 2 (tan x + tan y + tan z ) =2 (tan x + tan y + tan z )
(1) (2) (3)

Similarly sin 2 y u y =

sin 2 z u y =

Adding (1), (2) and (3) we get,

sin 2 x u x + sin 2y u y + sin 2z u z =

Thus sin 2x u x + sin 2y u y + sin 2z u z = 2

5.

If u = log (x3 + y3 + z3 3xyz) then prove that


-9 that x + y + z u = (x + y + z )2
2

3 u u u = + + and hence show x y z x + y + z

Solution : u = log (x3 + y3 + z3 3xyz) is a symmetric function


u 3x 2 - 3yz = 3 x x + y3 + z 3 - 3xyz u 3 y 2 - 3zx = 3 y x + y3 + z 3 - 3xyz u 3z 2 - 3xy = 3 z x + y3 + z 3 - 3xyz
Adding (1), (2) and (3) we get, 10 MAT11 92 (1)

(2)

(3)

Engineering Mathematics I

Dr. V. Lokesha

2011

u u u 3 x 2 + y 2 + z 2 xy yz zx + + = x y z x 3 + y3 + z 3 3xyz

)
)

Recalling a standard elementary result,


a 3 + b 3 + c 3 3abc = (a + b + c ) a 2 + b 2 + c 2 ab bc ca

We have,

u u u 3 x 2 + y 2 + z 2 xy yz zx + + = x y z ( x + y + z ) x 2 + y 2 + z 2 xy yz zx

Thus

u u u 3 + + = x y z x + y + z
2

Further x + y + z u

= x + y + z = x + y + z = x + y + z =
=

u x + y + z u u u x + y + z 3 x+ y+ z , by using the earlier result.

3 3 3 + + x x + y + z y x + y + z z x + y + z

(x + y + z)
2

(x + y + z)

(x + y + z)

( x + y + z )2

Thus

-9 x + y + z u = ( x + y + z )2

10 MAT11 93

Engineering Mathematics I

Dr. V. Lokesha

2011

6.

If u = f (r ) and x = r cos , y = r sin ,


2u 2u 1 prove that 2 + 2 = f (r ) + f (r ) r x y

Solution :Observing the required partial derivative we conclude that u must be a function of x, y. But
u = f( r) by data and hence we need to have r as a function of x, y. Since x = r cos , y = r sin we have x2 + y2 = r2.

we have u = f (r ) where r = x 2 + y2
2 u f (r ) 2 2 f (r ) = 3 r - x + 2 . x 2 and 2 x r r

f (r ) 2 2 u f (r ) 2 2 = + r y .y y 2 r3 r2

Adding these results we get,

2 u 2 u f (r ) f (r ) + = 3 2x2 - x2 + y2 + 2 x2 + y2 2 2 x y r r

)}

f (r ) 2 f (r ) 2 1 . r + 2 . r = f (r ) + f (r ) r r3 r

Thus 7.

2u 2u 1 + 2 = f (r ) + f (r ) 2 r x y

Prove that x

u u +y = nu x y

Proof : Since u = f (x, y) is a homogeneous function of degree n we have by the definition,


u = x n g ( y/x )

(1)

Let us differentiate this w.r.t x and also w.r.t.y


u y = x n . g ( y/x ) . - 2 + nx n - 1 g ( y/x ) x x
u = x n - 2 y g ( y/x ) + nx n - 1 g ( y / x ) x

ie.,

(2)

Also

u 1 = x n . g ( y/x ) . y x

10 MAT11 94

Engineering Mathematics I ie.,

Dr. V. Lokesha (3)

2011

u = x n - 1 . g ( y/x ) y u u +y as a consequence of (2) and (3) x y

Now consider x

= x - x n - 2 y g ( y/x ) + n x n - 1 g ( y/x ) + y x n - 1 g ( y / x ) = x n 1 y g ( y/x ) + n x n g ( y/x ) + x n - 1 y g ( y/x )

] [

= n . x n g ( y/x )

= n u, by using (1) Thus we have proved Eulers theorem

x
8.

u u +y = n u ; x u x + yu y = n u x y
2u 2u 2 y = n (n - 1) u + x x y x 2

Prove that x 2

Proof : Since u = f (x, y) is a homogeneous function of degree n, we have Eulers theorem

u u +y =nu x y

(1)

Differentiating (1) partially w.r.t. x and also w.r.t y we get,


2u u 2u u x 2 + 1. + y n = x x x y x

(2)

Also, x

2u 2u u u =n + y 2 +1 . y x y y y

(3)

We shall now multiply (2) by x and (3) by y.

x2

u 2u 2u u x x y and =nx + + 2 x xy x x
2u u u 2u + y2 2 + y = ny y x y y y

xy

Adding these using the fact that

2u 2u = we get, y x y x

10 MAT11 95

Engineering Mathematics I

Dr. V. Lokesha

2011

2 2 2u u u 2u u u 2 u x + 2 y y y n y + + = + + x x x 2 2 x xy y y y x x

ie., x 2

2 2u 2u 2 u + 2 y + y + n u = n (n u ), by using (1) x xy x 2 y 2

2 2u 2u 2 u +2xy +y + n (nu ) - nu = n (n - 1) u or x xy x 2 y 2 2 2 2u 2u 2 u +2xy +y + n (n - 1) u Thus x xy x 2 y 2 2

ie., x 2 u xx + 2 x y u xy + y 2 u yy = n (n - 1) u 9. If u =

u u u x y z + + +y +z =0 show that x x y z y+z z+ x x+y

Solution : (Observe that the degree is 0 in every term)


u= x y z + + y+ z z+ x x+ y

We shall divide both numerator and denominator of every term by x.

u=

1 y/ x z + + = x {g ( y/x, z/x )} y/x + z/x z / x + 1 1 + y / x


u is homogeneous of degree 0. n = 0

We have Eulers theorem, x Putting n = 0 we get, x 10.

u u u =nu +y +z z x y

u u u =0 +y +z z x y

x4 + y4 u u + x If u = log show that y =3 x+ y x y


4 x4 + y4 x4 1 + y4 / x4 3 1 + ( y/x ) e = = =x x+ y x (1 + y/x ) 1 + (y / x ) u

Solution : we cannot put the given u in the form xn g (y/x)

ie., eu = x3 g (y/x) eu is homogeneous of degree 3 n = 3 Now applying Eulers theorem for the homogeneous function eu 10 MAT11 96

Engineering Mathematics I

Dr. V. Lokesha

2011

We have x ie., x e u

eu eu +y = n eu x y

( )

( )

u u + y eu = 3e u x y u u +y =3 x y

Dividing by eu we get x 11.

x3 + y3 If u = tan -1 x y show that


(i) x ux + y uy = sin 2 u (ii) x2uxx + 2 x y uxy + y2uyy = sin 4 u sin 2 u

x 3 + y3 Solution : (i) u = tan x - y by data


-1

tan u =

3 x3 + y3 x3 1 + y3 / x3 1 + ( y/x ) = = x2 x-y x (1 - y/x ) 1 ( y / x )

ie., tan u = x2/g (y/x) tan u is homogeneous of degree 2. Applying Eulers theorem for the function tan u we have,

(tan u ) (tan u ) +y = n . tan u ; n = 2 x y u u + y sec 2 u = 2 tan u x y

ie., x sec 2 u or

u u 2 tan u sin u +y = = 2 cos 2 u = 2 cos u sin u = sin 2 u 2 x y cos u sec u

xu x + yu y = sin 2u

(ii)

We have xux + y uy = sin 2 u

(1) (2) (3)

Differentiating (1) w.r.t x and also w.r.t y partially we get


x u xx + 1 . u x + yu xy = 2 cos 2u . u x

And x uyx + yuyy + 1 . uy = 2 cos 2u . uy Multiplying (2) by x and (3) by y we get,


x 2 u xx + x u x + xy u xy = 2 cos 2 u. xu x

10 MAT11 97

Engineering Mathematics I
xy u yx + y 2 u yy + y u y = 2 cos 2 u. yu y

Dr. V. Lokesha

2011

Adding these by using the fact that uyx = uxy, we get


x 2 u xx + 2 x y u xy + y 2 u yy + xy x + yu y = 2 cos 2u xu x + y u y

By using (1) we have,


x 2 u xx + 2 x y u xy + y 2 u yy = 2 cos 2u sin 2 u - sin 2u

(since sin 2 = 2 cos sin , first term in the R.H.S becomes sin 4u) Thus x2uxx + 2 x y uxy + y2 uyy = sin 4 u sin 2u 12.

x y z u u u If u = f y, z, x Prove that x x + y y + z z = 0
>> here we need to convert the given function u into a composite function. Let u = f (p, q, r ) where p =

x y z ,q= ,r = x y z

ie., {u (p, q, r ) (x, y, z )} u x, y, z

ie.,

u u p u q u r = + + x p x q x r x
1 u u u u z . . 0+ .- = + y r x 2 x p q

u x u z u = x y p x r

(1)

Similarly by symmetry we can write,

y z

u y u x u = y z q y p u z u y u = z x r z q u u u =0 +y +z z x y

(2) (3)

Adding (1), (2) and (3) we get x

10 MAT11 98

Engineering Mathematics I 13. If u = f(x y, y z, z x) show that

Dr. V. Lokesha

2011

u u u + + =0 x y z

>> Let u = f(p, q, r) where p = x y, q = y z, r = z x

ie.,

u u p u q u r = + + x p x q x r x u u u u (- 1) = .1+ .0 + x p q r u u u = x p r
(1)

Similarly we have by symmetry

u u u = y q p u u u = z r q
Adding (1), (2) and (3) we get,

(2) (3)

u u u + + x y z

=0

14.

If z = f(x, y) where x = r cos and y = r sin


1 z z z Show that + = + 2 r x y r
2 2 2

Solution : {z ( x, y ) (r, ) } z (r, )

ie., and or

z z x z y z x z y = ; + + r x r y r x y z z z = cos + sin r x y
(1)

z z ( - r sin ) + z (r cos ) = r - z sin + z cos = x y y x

1 z z z sin + = cos = r x y

squaring and adding (1), (2) and collecting suitable terms have, 10 MAT11 99

Engineering Mathematics I

Dr. V. Lokesha
2

2011

1 z + 2 r r
2

z z 2 2 = cos + sin x

z z z z z 2 2 + y sin + cos + 2 x y cos sin - 2 x y sin cos z r


2

1 z z z = + ie., R.H.S = L.H.S r 2 x y


2 2

15.

If z = f (x, y ) where x = e u + e v , y = e -u e v Prove that x

z z z z y = x y u v

Solution : {z (x, y ) (u, v )} z (u, v )


z z x z y z z x z y = + ; = + u x u y u v x v y v z z z = . eu + - e -u u x y

ie.,

( )

(1) (2)

z z - v z = -e + - e-v v x y
Consider R.H.S =

( )

( )

z z and (1) (2) yields u v

z z z u z - u e + e-v e ev = .x. y x y x y
Thus 16. Find

z z z z =x y y u y x

ie., R.H.S = L.H.S

(u , v, w ) where u = x2 + y2 + z2, v = xy+yz+zx, w=x+y+z (x , y, z )

Solution :

u x (u , v, w ) v The definition of J = = x (x , y, z ) w x

u y v y w y

u z v z w z

But

u = x2 + y2 + z2, v = xy+yz+zx, w=x+y+z 10 MAT11 100

Engineering Mathematics I

Dr. V. Lokesha

2011

Substituting for the partial derivatives we get

2x 2y 2z J= y+z x+z y+x 1 1 1

Expanding by the first row, J = 2x {(x + z) (y + x)} -2y {(y + z) (y + x)} +2z {(y + z) (x + z)} = 2x (z-y) 2y(z-x) + 2z(y-x) = 2(xz xy yz + xy + yz xz) = 0 Thus J = 0 17. If u =
(u , v, w ) zx xy yz ,v= ,w= , show that =4 y x z (x , y, z )

Solution :

by data u =

zx xy yz ,v= ,w= y x z

(u , v, w ) = (x , y, z )

u x v x w x

u y v y w y

u yz z x2 v z = z y w y z z

z x zx y2 x z

y x x y xy z2

yz x2

zx xy x x y y2 z2 z
-

z z zx y x y z x y zx 2 + 2 x y y z y x y z z y
x y

yz x 2 x 2 z x x y x = 2 + + x yz yz x z z x y
= 0+1+1+1+1=4
(u , v, w ) =4 (x , y, z )

Thus

10 MAT11 101

Engineering Mathematics I 18.

Dr. V. Lokesha

2011

If u + v = ex cos y and u v = ex sin y find the jacobian of the functions u and v w.r.t x and y.

Solution :

u (u , v ) x we have to find = v (x , y ) x

u y v y

Using the given data we have to solve for u and v in terms of x and y. By data u + v = ex cos y u v = ex sin y (1) + (2) gives : 2 u = ex (cos y + sin y) (2) (2) gives : 2 v = ex (cos y sin y) Ie., u = (1) (2)

ex ex (cos y + sin y) ; v = (cos y sin y) 2 2

ex v e x u = (cos y + sin y), = (- sin y - cos y) x 2 x 2


ex ( sin y + cos y) 2 ex (sin y + cos y) 2

ex (cos y + sin y) (u , v ) Now = 2 ex ( x, y ) (cos y sin y) 2

= =

ex ex . { - ( cos y + sin y)2 (cos y sin y)2} 2 2 e2x e2x {1+sin 2y) + (1 sin 2y)} = 4 2
(u , v ) e 2 x = 2 (x , y )

Thus

10 MAT11 102

Engineering Mathematics I

Dr. V. Lokesha

2011

19.

(a) If x = r cos , y = r sin find the value of (b) Further verify that

(r, ) (x , y )

(x , y ) (r, ) . =1 (r, ) (x , y )

(a)

Solution :

We shall first express r, in terms of x and y.

We have x = r cos , y = r sin by data.

x2 + y2 = r2 and

y = tan or = tan-1 (y/x) x

Consider r2 = x2 + y2 Differentiating partially w.r.t x and also w.r.t y we get, 2r


r = 2x x r = 2y y

and and

2r

r x = x r

r y = y r

Also consider = tan 1 ( y / x )

i.e.,

1 y = . 2 2 x 1 + (y / x) x y = 2 x x + y2

and and

1 1 = . 2 y 1 + (y / x) x x = 2 y x + y2

r r x (r , ) r dy r = = Now y ( x, y ) 2 x + y2 dx dy

y r x 2 x + y2

i.e.,

x2 y2 (x2 + y 2 ) 1 + = = r(x 2 + y 2 ) r(x 2 + y 2 ) r(x 2 + y 2 ) r

(r , ) 1 = ( x, y ) r

10 MAT11 103

Engineering Mathematics I

Dr. V. Lokesha

2011

Solution (b) : Consider x = r cos , y = r sin


x ( x, y ) r = (r , ) y r
( x, y ) =r (r , )
( x, y ) (r , ) 1 = r =1 r ( x, ) ( x, y )

x cos r sin = = r (cos 2 + sin 2 ) = r y sin r cos d

From (1) and (2) :

20.

If x = u (1 v ), y = uv then show that JJ / = 1

Solution : x = u (1 v); y = uv
y x = (1 v), =v u u y x = u , =u v v

x x ( x, y ) u v (1 v) u = J= = v u (u, v) y y u v
= (1 v)u + uv = u J =u

Next we shall express u and v in terms of x and y. By data x = u - uv and y = uv

Hence x + y = u. Also v =

y y = u x+ y y x+ y
u u = 1, = 1, x y

Now we have, u = x + y; v =

10 MAT11 104

Engineering Mathematics I
x v ( x + y ) 0 y 1 = = x ( x + y )2 ( x + y) 2

Dr. V. Lokesha

2011

u (u , v) x = J/ = ( x, y ) v x
=

u 1 y = y v ( x + y) 2 y

x ( x + y) 2

y x+ y x 1 1 + = = = 2 2 2 ( x + y) u ( x + y) ( x + y) ( x + y)
1 1 Hence J J / = u Thus JJ / = 1 u u

Thus J / = 21.

State Taylors Theorem for Functions of Two Variables.

Statement: Considering f (x + h, y + k) as a function of a single variable x, we have by Taylors Theorem f (x + h, y + k) = f (x, y + k) + h

f ( x, y + k ) h 2 2 f ( x, y + k ) + + ---------(1) x 2! x 2

Now expanding f (x, y + k) as function of y only, f (x, y + k) = f (x, y) + k

f ( x, y ) k 2 2 f ( x, y ) + -------+ 2! y y 2 f ( x, y ) k 2 2 f ( x, y ) +-------------------------------- + + 2! y y 2

(i) takes the form f (x + h, y + k) = f (x,y) + k


h

f ( x, y ) k 2 2 f ( x, y ) + + f ( x, y ) + k 2 x y 2! y
+

h2 2 ( x, y ) f ( x, y ) + k + 2 2! x y
2 f f 1 2 2 f 2 f 2 f h hk k +k + + 2 + 2 x y 2! xy y 2 x

Hence f (x + h, y + k) = f (x , y) +h In symbol we write it as

+ (1)

10 MAT11 105

Engineering Mathematics I

Dr. V. Lokesha
2

2011

1 F (x + h, y + k) = f (x,y) + h x + k y f + 2! h x + k y f +
Taking x = a and y = b, (1) becomes f (a + h, b + k) = f (a,b) + [h f x (a,b) + kf y (a,b)] +
1 [h 2 f xx (a,b)+ 2hkf xy (a,b) 2!

+ k 2 f yy (a,b)] + --------Putting a + h = x and b + k = y so that h = x a, k = y b, we get F (x,y) = f (a,b) + [(x a) f x (a,b) + (y b) f y (a,b)] =
1 [(x a)2 f xx (a,b) + 2 (x a) (y b) f xy (a,b) + (y b)2 f yy (a,b)] +------ (2) 2!

This is Taylors expansion of f (x,y) in powers of (x a) and (y b). It is used to expand f (x,y) in the neighborhood of (a,b) corollary, putting a = 0, b = 0 in (2), we get f (x,y) = f (0,0) + [x f x (0,0) + y f y (0,0)] + This is Maclaurins Expansion of f (x,y) 22.
1 [ x 2 f xx (0,0) + 2xy f xy (0,0) 2! + y 2 f yy (0,0) ] + --------- (3)

Expand e x log (1 + y) in powers of x and y up to terms of third degree.

Solution: Here f (x,y) = e x log (1 + y)

f (0,0) = 0

f x (x,y) = e x log (1 + y) f x (0,0) = 0 f y (x,y) = e x

1 1+ y

f y (0,0) = 1

f xx (x,y) = e x log (1 + y) f xx (0,0) = 0 f xy (x,y) = e x

1 1+ y

f xy (0,0) = 1

10 MAT11 106

Engineering Mathematics I f yy (x,y) = - e x (1 + y) -2

Dr. V. Lokesha

2011

f yy (0,0) = -1

f xxx (x,y) = e x log (1 + y) f xxx (0,0) = 0 f xxy (x,y) = e


x

1 1+ y

f xxy (0,0) = 1

f xyy (x,y) = -e x (1 + y) 2 f xyy (0,0) = -1 f yyy (x,y) = 2e x (1 + y)-3 f yyy (0,0) = 2

Now, Maclaurins expansion of f (x,y) gives 1 f (x,y) = f (0,0) + x (f x (0,0) + y fy (0,0) + {x2 fxx (0,0) + 2xy fxy (0,0) + 2! 1 y2 fyy (0,0)} + {x3 fxxx (0,0) + 3x2 y fxxy (0,0) + 3xy2 fxyy (0,0) + y3 fyyy (0,0)} + ----------2! ex log (1 + y) = 0 + x.0 + y (1) +
+
1 {x2.0 + 2xy (1) + y2 (-1)} 2!

1 3 {x .0 + 3x2y (1) + 3xy2 (-1) + y3(2)}+--------2! 1 1 1 = y + xy - y2 + (x2y = xy2) + y3 + ---------2 2 2

23.

Expand f (x,y) = ex Cosy by Taylors Theorem about the point 1, up to the Second 4 degree terms. f = 1, = e Solution: f (x,y) = ex Cosy and a = 1, b = 2 4 4 e fx (x,y) = ex Cos y f 1, = 2 4 e fy (x,y) = -ex Sin y fy 1, = 2 4 e fxx(x,y) = ex Cos y fxx 1, = 2 4 e fxy (x,y) = -ex Sin y fxy 1, = 2 4 e fyy 1, = fyy (x,y) = - ex Cos y 2 4 Hence by Taylors Theorem, we obtain

10 MAT11 107

Engineering Mathematics I

Dr. V. Lokesha

2011

f (x,y) = f 1, + ( x 1) f x + y fy + 4 4 2 1 2 ( 1 ) 2 ( 1 ) f x + x y f yy + ------------ x 4 2!
i.e., ex Cosy =

2 ( x 1)
ex

e e 1 + y + ( x 1) + 4 2 2 2! 2 2 e e e + 2( x 1) y + -----------+y 4 4 2 2 2
e

2 e 1 2 Cosy = 1 + ( x 1) y 4 + 2! ( x 1) 2( x 1) y 4 y 4 + -------} 2

Exercise: 1) Expand exy up to Second degree terms by using Maclaurins theorem 2) Expand Log (1 x y ) up to Third degree terms by using Maclaurins theorem 3) Expand x2y about the point (1,-2) by Taylors expansion 4) Obtain the Taylors expansion of ex Siny about the point 0, 5) Expand esinx up to the term containing x4

) up to Second degree terms

10 MAT11 108

Engineering Mathematics I

Dr. V. Lokesha

2011

Maxima and Minima:In mathematics, the maximum and minimum (plural: maxima and minima) of a function, known collectively as extrema (singular: extremum), are the largest and smallest value that the function takes at a point within a given neighborhood. A function f (x, y) is said to have a Maximum value at (a,b) if their exists a neighborhood point of (a,b) (say (a+h, b+k)) such that f (a, b) > f (a+h, b+k). Similarly, Minimum value at (a,b) if there exists a neighborhood point of (a,b) (say (a+h, b+k)) such that f (a, b) < f (a+h, b+k). A Minimum point on the graph (in red) f ( x, y ) = x 2 + y 2 (1 x)3

A Maximum point on the graph is at the top (in red)

10 MAT11 109

Engineeri ing Mathem matics I

Dr. V. Lokesha

2011

ddle point on n the graph of z=x2y2 (in red) A sad

Saddle point betw ween two hills s.

Nec cessary an nd Suffic cient Con ndition: If fx =0 and fy =0 (Neces ssary Cond dition) Functi ion will be minimum m if AC-B2 > 0 and A > 0 Functio on will be maximum m if AC-B2 > 0 and A < 0 Functio on will be neither n ma axima nor minima m if AC-B2 < 0 If AC C-B2 = 0 we w cannot make m any conclusion n without any a further ana alysis

w where A = f xx , B = f xy , C = f yy

10 MA AT11 110 0

Engineering Mathematics I

Dr. V. Lokesha

2011

Working Procedure:First we find Stationary points by considering fx =0 and fy =0 . Function will be minimum if AC-B2 > 0 and A > 0 at that stationary point Function will be maximum if AC-B2 > 0 and A < 0 at that stationary point Function will be neither maximum nor minimum if AC-B2 < 0 at that stationary point and it is called as SADDLE POINT.
25. Explain Maxima & Minima for Functions of Two Variables& hence obtain the Necessary Conditions for Maxima, Minima. Solution: Let Z = f (x,y) be a given function of two independent variables x & y. The above equation represents a surface in 3D. Z P(a,b) f (a,b) Z = f (x,y) A = f (a,b) N X a O b M(a,b) Y

A given points (a,b) on the surface has Co-ordinates [a,b, f (a,b)]

10 MAT11 111

Engineering Mathematics I

Dr. V. Lokesha

2011

Definition: The function Z = f (x,y) is said to be a maximum at the point (a,b) if f (x,y) < f (a,b) in the neighborhood of the point (a,b) Z Cap

Cup O Y (a,b) Definition: X

The function Z = f (x,y) is said to posses a minimum at the point (a,b) if f (x,y) > f (a,b) in the neighborhood of the point (a,b)
Necessary Condition for Maxima, Minima:

If Z = f (x,y) has a max or min at (a,b) then f x (a,b) = 0, fy (a,b) = 0


Sufficient Conditions for Maxima, Minima: Put R = fxx (a,b), S = fxy (a,b), T = fyy (a,b) (1) Suppose S2 RT > 0

There is no maxima or minima at (a,b)


(2) Suppose S2 RT < 0 Thus there is maxima or minima at (a,b) according as R < 0 Or R > 0 (3) Suppose S2 RT = 0, Then there is a saddle point at (a,b)

10 MAT11 112

Engineering Mathematics I

Dr. V. Lokesha

2011

26. Find the maxima and minima of the functions f (x,y) = x3 + y3 3axy, a > 0 is constant. Solution: Given f (x,y) = x3 + y3 3axy fx = 3x2 3ay, fy = 3y2 3ax fxx = 6x fyy = 6y. Put fx = 0, fy = 0 and solve i.e., 3x2 3ay = 0 & 3y2 3ax = 0 i.e., x2 = ay & y2 = ax

x2 y= a

x2 a = ax

(x2 = ay)

x4 = ax a2

x4 = a3x
i.e., x (x3 a3) = 0

x = 0, x = a y = 0, y = a The critical Or stationary points are (0,0), (a,a) and (a,-a)


(1) At (0,0) R = fxx (0,0) = 0 S = fxy (0,0) = -3a T = fyy (0,0) = 0

S2 RT = 9a2 0 = 9a2 > 0 There is neither a maximum or a minimum at (0,0)

10 MAT11 113

Engineering Mathematics I

Dr. V. Lokesha

2011

27. Examine the following functions for extreme values f = x4 + y4 2x2 + 4xy 2y2 Solution: fx = 4x3 4x + 4y fy = 4y3 4x 4y fxy = 4, fxx = 12 x2 4, fyy = 12y2 4 Put fx = 0, fy = 0 and solve i.e., 4x3 4x + 4y = 0 4y3 + 4x 4y = 0 Adding (1) & (2), we get 4 (x3 + y3) = 0 i.e., x3 + y3 = 0 i.e., y = - x Substitute y = -x in (1), we get 4x3 4x 4x = 0 i.e., 4x3 8x = 0 i.e., x3 2x = 0 x (x2 2) = 0 i.e., x = 0 & x2 2 =0 i.e., x = 0 & x = x=

(1) (2)

2,-

2 2 ,2 and corresponding values of y are y = 0, 2 ,- 2


, - 2,

x = 0,

2,

The critical points are (0,0),


(1) at (0,0) R = fxx (0,0) = - 4

10 MAT11 114

Engineering Mathematics I

Dr. V. Lokesha

2011

S = fxy (0,0) = 4 T = fyy (0,0) = -4

S2 RT = 16 (-4) (-4) = 16 16 = 0
i.e., S2 RT = 0, These is a saddle point at (0,0) (2) at

2,- 2

R = fxx S = fxy T = fyy

2,- 2 2 ,- 2 2 ,2

=24 4 = 20 =4 = 20

S2 RT = 16 (20) (20) = 16 400 = -384 < 0


Thus these is neither maximum nor minimum according to R < 0 or R > 0 at Hence R = 20 > 0

2,-

There is a minimum at
f min =

2 ,4

2
2

( 2 ) + ( 2 )
4

2 2

( )

+4 2 2 2 2

) (

=4+4484 =-8 (3) at - 2 , 2 R = fxx 2 , 2 = 20 > 0 S = fxy 2 , 2 = 4 T = fyy 2 , 2 = 20 S2 RT = 16 400 = -384 < 0 10 MAT11 115

Engineering Mathematics I Since R > 0, There is minima at 2 , 2

Dr. V. Lokesha

2011

) )

fmin = - 8 at 2 , 2

) ( ) (

Extreme Value = - 8 at 2 , 2 & 2 , 2


Exercise:

1) Find the extreme values of f = x3 y2 (1 x y) 2) Determine the maxima or minima of the function Sin x + Sin y + Sin (x + y) 3) Examine the function f(x,y) = 1+ sin( x2 + y2) for extremum. 28. If PV2 = K and if the relative errors in P is 0.05 and in V is 0.025 show that the error in K is 10%.

Solution : PV 2

K by data. Also

P
P

= 0.05

and

V
V

= 0.025

log P + 2 log V = log K

(log P) + 2 (log V ) = (log K )

1 1 1 P + 2 V = K p V K K K i.e., 0.05 + 2(0.025) = or = 0.1 K K K 100 = (0.1) 100 = 10 K i.e.,


Thus the error in K is 10%.

10 MAT11 116

Engineering Mathematics I 29.

Dr. V. Lokesha

2011

The time T of a complete oscillation of a simple pendulum is given by the formula


T = 2 l / g

(i) (ii)

If g is a constant find the error in the calculated value of T due to an error of 3% in the value of l. Find the maximum error in T due to possible errors upto 1% in l and 3% in g.

Solution :
l
l

(i )

T = 2 l / g ,

g = Constant,

100 = 3

1 (log l log g ) 2 1 (log T ) = (log 2 ) + (log g ) 2 1 l T i.e., = 0+ 0 T 2 l 1 l T 1 or 100 = 100 = (3) = 1.5 T 2 l 2 log T = log 2 +

the error in T = 1.5%.


(ii) If g is not a constant we have,

T
T

100 =

1 l 1 g 100 100 2 l 2 g

The error in T will be maximum if the error in l is positive and the error in g is negative (or viceversa) as the difference in errors converts in to a sum.
1 T 1 100 = (+1) (3) = 2 max 2 T 2

the maximum error in T is 2%.


10 MAT11 117

Engineering Mathematics I

Dr. V. Lokesha

2011

30.

The current measured by a tangent galvanometer is given by the relation c = k tan where is the angle of deflection. Show that the relative error in c due to a given error in is minimum when = 45 0.

Solution : Consider c = k tan . K is taken as a constant.


log c = log k + log (tan ) (log c) = (log k ) + log (tan )

1 sec 2 i.e., c = 0 + tan c

i.e.,

c
c

cos 1 sin cos 2

or

sin cos

i.e.,

c
c

2 sin 2

The relative error in c being c / c minimum when the denominator of the R.H.S. is maximum and the maximum value of a sine function is1.
sin 2 = 1 2 = 90 0 or = 45 0

Thus the relative error in c is minimum when = 450

31.

If T =

1 mv 2 is the kinetic energy, find approximately the change in T as m changes 2

from 49 to 49.5 and v changes from 1600 to 1590.

6 Marks

Solution : We have by data T = mv 2 and


m = 49, m + m = 49.5 m = 0.5 v = 1600, v + v = 1590 v = 10
10 MAT11 118

1 2

Engineering Mathematics I We have to find T .


T = 1 ( mv 2 ) 2

Dr. V. Lokesha (logarithm is not required)

2011

1 m( 2vv) + m.v 2 2 i.e, =

} }

1 ( 49) ( 2) (1600 ) ( 10) + (0.5) (1600 ) 2 = 1,44,000 2

Thus the change in T = T = 1,44,000 32. The pressure p and the volume v of a gas are concentrated by the relation
pv 1.4 = cons tan t. Find the percentage increase in pressure corresponding to a

diminution of % in volume.

Solution :
pv1.4 = Constant = c( say ), by data.

log p + 1.4 log v = log c (log p) + 1.4 (log v) = (log c)


i.e,

v 1 v + 1.4 = 0; But 100 = , by data. p v 2 v

p v 100 + 1.4 100 = 0 or 100 = +0.7 . p p v

Thus the percentage increase in pressure = 0.7 33. Find the percentage error in the area of an ellipse when an error of +1% is made in measuring the major and minor axis.

Solution : For the ellipse x 2 / a 2 + y 2 / b 2 = 1 the area (A) is given by ab where 2a and 2b are
the lengths of the major and minor axis. Let 2a = x and 2b = y. By data

x
x

100 = 1,

y
y

100 = 1.

10 MAT11 119

Engineering Mathematics I
A = ab = x y = xy 2 2 4

Dr. V. Lokesha

2011

log A = log ( / 4) + log x + log y (log A) = log ( / 4) + (log x) + (log y )


i.e.,

A
A

=0+

x
x

y
y

or

A
A

100 =

x
x

100 +

y
y

100

A
A

100 = 1 + 1 = 2

Thus error in the area = 2% 27. If the sides and angles of a triangle ABC vary in such way that the circum radius remains constant, prove that

a
cos A

b
cos B

c
cos c

=0

Solution : If the triangle ABC is inscribed in a circle of radius r and if a,b,c respectively
denotes the sides opposite to the angles A,B,C we have the sine rule (formula) given by
a b c = = = 2r sin A sin B sin C

or a = 2r sin A, b = 2r sin B, c = 2r sin C

a = 2r (sin A), b = 2r (sin B), c = 2r (sin C )

i.e., a = 2r cos AA, b = 2r cos BB, c = 2r cos CC


or

a
cos A

= 2rA,

b
cos B

= 2 r B ,

c
cos C

= 2 rC

Adding all these results we get,

a
cos A

b
cos B

c
cos C

= 2r (A + B + C ) = 2r ( A + B + C )

But A+B+C = 180 = radians = constant.

( A + B + C ) = (constant) = 0
Thus

a
cos A

b
cos B

c
cos C

=0

10 MAT11 120

Engineering Mathematics I

Dr. V. Lokesha

2011

Multiple choice Questions:


DIFFERENTIAL CALCULUS 1) The radius of curvature at any point of catenary S=C tan is a) c sec2 b) c cos2 -2 c) c tan d)none 2) Stationary points of f(x,y) = sin x+ sin y+ sin ( x + y ) is a) (/3, /3) b) (/6, /6) c) (/4, /4) d) none 3) If the curvature is zero, that point is known as-------------------a) Point of inflection b) Stationary point c) (a) or (b) d) none 4) The radius of curvature of the curve y = 4 sin x- sin 2x at x=/2 is a) 55/4 b) -55/4 c)55/2 d) none 5) The function x2+2xy+ 2y2+2x+2y has a minimum value at a) (-3/2,1/2) b)(3/2, 1/2 ) c) (3/2,-1/2) d)none

6) The stationary point of f(x,y,z)=x2+y2+z2 where x,y,z are connected by x+y+z=a is a) (a,a) b) (-a,-a) c) (2a, 0) d) none 7) The radius of curvature of the curve x+y =1 at the point (1/4,1/4) is -----------a) =1/2 b) =2 c) = -1/2 d) none ) 8) The expression for the derivative of arc length in Cartesian form is given by a) ds/d =
r 2 + ( dr/d ) 2

b) ds/dx=

1 + (dy/dx) 2

c) ds/dr= 1 + ( d?/dr) 2

d) ds/dt= 1 + (dy/dx)2

9)The formulae for radius of curvature in Cartesian form is a) = (x1) 2 + (y1) 2)3/2 /x1y11+x11y1 b) =

(1+ y )
2 1

3/ 2

y2

c) = [(x1) 2 + (y1) 2)3/2 ] 10 MAT11 121

Engineering Mathematics I d) = (1+(y1) 2 ) 3/2 10) The function for which Rolles theorem is true is: a) f(x)=log x in the interval [1/2,2] b) f(x)=|x+1| in the interval [-2,2] c) f(x)=| x |in the interval [-1,1] d) Non of the above

Dr. V. Lokesha

2011

11) The value of cin Rolles theorem ,where / 2 < c < / 2 and f(x)= cosx is equal to: a /4 b) / 3 c) d)0 12).The expansion of tanx in powers of x by Maclaurins theorem is valid in the interval : d) ( / 2, / 2) a) ( , ) b) ( 3 / 2,3 / 2) c) ( , ) 13) The value of cin Lagrange,s mean value theorem ,where [1,2] and f(x)= x(x-1) is : a)5/4 b)3/2 c)7/4 d) 11/6 14) The value of cin Rolles theorem, where [0, ] and f(x)= sinx is equal to: a) / 6 b) / 3 c) / 2 d) Non of these 15) The maximum value of logx/x is: a) 1 b)e c)2/e d) 1/e 16) The maximum value of (l/x ) x is equal to: a)e b)1 c) e 1/e d) (1/e)e 17) The difference bwtween the maximum and minimum values of the function a sinx+bcosx is: a) 2 a 2 + b 2 b) 2(a 2 + b 2 ) c) a 2 + b 2 d) a 2 + b 2

18) Which one of the following statement is correct for the function f(x)=x3 a) f(x) has a maximum value at x=0 b) f(x) has a manimum value at x=0 c) f(x) has a neither a maximum nor a manimum value at x=0 d) f(x) has no point of inflexion 19) which one of the following is not an indeterminate form a) + b) c) / 20) In Lagrange,s mean value theorem , f1(c)= a) f(a) f(b)/ b -a b) f(b) f(a) / a -b c) f(b) f(a) / b -a d) none 10 MAT11 122 d)

0 X

Engineering Mathematics I

Dr. V. Lokesha

2011

21) In Cauchys mean value theorem , f1(c) / g1(c) = a) f(a) f(b)/ g(b) g(a) b) f(b) f(a) / g(b) g(a) .c) f(b) +f(a) / g(b) +g(a) d) none x x a - b 22) lim is equal to : x 0 x a) 0 b) c) log (a/b) d) log (a-b)

23) The value of lim


x0

log x is equal to: x 1

a) -1

b)

c) 1
x 0

d) 0

24) The value of lim (1 cos x / 3 x2 )is equal to: a) 3


x0

b) 1/3

c) 1/6

d) 1/9

25) lim ( ex + e-x 20) / x2 is equal to : a) 1 b) -1 c) 1/2


x0

d) -1/2

26) The value of lim ( 1 + x )1/x is : a) 1 b) -1 c) 1/e


lim
x 0

d) e
sin x =. x

27) The value of

d)0 a) 1 b) 2 c) 3 28) The formulae for radius of curvature in polar form is a) = rdr/dp c) =(r 2 + (r1) 2)3/2 b) = [ (r 2 + (r1) 2)3/2] / [r2+2 ( r1) 2- r r2] d) None

29) The value of lim


x 0

1 1/x = b) c) - d) 0 10 MAT11 123

a) 1

Engineering Mathematics I

Dr. V. Lokesha

2011

KEY ANSWERS:

1-a 11-d 21-b

2-a 12-d 22-c

3-a 13-b 23-c

4-a 14-c 24-c

5-a 15-d 25-a

6-a 16-c 26-c

7-a 17-a 27-a

8-b 18-c 28-b

9-b 19-a 29-b

10-a 20-c --

10 MAT11 124

You might also like